Frs Key

You might also like

Download as pdf or txt
Download as pdf or txt
You are on page 1of 34

II INDIAN POLITY AND GOVERNANCE II

SOLUTIONS
providing special ramps for disabled people forms part of
Fundamental Rights affirmative action and does not come under formal equality.
1. Solution: (d) Pair 3 is correctly matched. In India, many face lack of equal
Exp) Option dis the correct answer. opportunities from both lack of facilities and prevailing
customs in the society. Women, for instance, may not enjoy
Statement a is correct. Arbitrariness denotes
equal rights of inheritance in some groups, or there may
unreasonableness and is against all norms of law and justice.
be social prohibitions regarding their taking part in certain
The officers have to carry out the law made by Parliament and
kinds of activities, or they may even be discouraged from
should not exercise any more power. Any wide discretionary
obtaining higher education. The state should make policies
power or arbitrary power vested in the State violates Rule of
to prevent discrimination or harassment of women in public
Law. Any unreasonable or arbitrary action is considered to
places or employment, to provide incentives to open up
be illegal. Therefore, it can be nullified by a court of law
education or certain professions to women, and other such
Statement b is correct. As per UN, rule of law is a principle measures. Thus, providing equal inheritance rights to
of governance in which all persons, institutions and entities, women is providing equality of opportunities.
public and private, including the State itself, are accountable
to laws. These laws need to be in consistent with international 3. Solution: (c)
human rights norms and standards. Liberty and civil rights Exp) Option c is the correct answer.
are considered as the main feature of the rule of law. Statement l is correct. Marxism argues that the root cause of
Statement c is correct. One of the essences of rule of law is entrenched inequality was private ownership of important
that the citizens should be able to enforce the rights that are economic resources such as oil, or land, or forests, as well
recognised and the courts must be sufficiently empowered as other forms of property. It points out that such private
for that purpose. It is said that, to promote the supremacy ownership did not only make the class of owners wealthy, it
of law, a system of strong, independent and impartial courts also gave them political power. Thus, there should be public
is mandatory. For this, the courts should have the power of ownership of important resources and property.
judicial review. Statement 2 is correct. Liberalism upholds the principle of
Statement d is incorrect. Administrative Law deals not only competition as the most efficient and fair way of distributing
with conferring of powers on the officers, but also focuses on resources and rewards in society. They believe that while
controlling the powers of the government. It is Administrative states may have to intervene to try and ensure a minimum
Law that makes it possible for the Courts to strike down any standard of living and equal opportunities for all, this cannot
arbitrary exercise of power by the Government. Thus, Indian by itself bring equality and justice to society. Competition
Administrative Law has benefited from Rule of Law by way between people in free and fair conditions is the most just
of establishing judicial review of administrative action to and efficient way of distributing rewards in a society.
ensure that administration acts in accordance with law. Statement 3 is correct. One common agreement between
Gandhism and Marxism is the final goal of stateless and
2. Solution: (a)
classless society, whereas means to achieve these final goals
Exp) Option a is the correct answer. differ. For Marx, the State is an instrument of oppression
Pair 1 is incorrectly matched. Affirmative actions are taken and an organ of the bourgeoisie that only works for
to eliminate deep rooted social inequalities. Sometimes it maintaining class dominance. Similarly for Gandhi, in an
is necessary to treat people differently in order to ensure that ideal state, there is no State. In the ideal state, there is no
they can enjoy equal rights. For instance, disabled people political power because there is no State.
may justifiably demand special ramps in public spaces so
4. Solution: (b)
that they get an equal chance to enter public bmldmgs.
These should not be seen as infringements of equality but Exp) Option b is the correct answer.
as enhancement of equality. Thus, providing free primary Statement a is correct: According to libertarians, if the state
healthcare services to all is not affirmative action. tries to provide equality, then it may result in the increased
Pair 2 is incorrectly matched. Social, economic and interference in individual liberty. More focus on equality
political inequalities all over the world have been protected leads to distribution of resources which may result in loss of
by customs and legal systems that prohibited some sections individual liberty.
of society from enjoying certain kinds of opportunities and Statement b is incorrect: More liberty does not always
rewards. Formal equality requires that the government and result in more economic equality as all people does not have
the law of the land should stop protecting these systems same potential and opportunities. This may result in high
of inequality. The Constitution prohibits discrimination on inequality in society. Example - rising inequality in highly
grounds of religion, race, caste, sex or place of birth. Thus, liberal western societies.

Practicebook·--------------------------------------------IJD

KING R QUEEN P [ऋषभ राजपूत ]


II INDIAN POLITY AND GOVERNANCE II
Statement c is correct: Liberty and equality are not always Article 25 - Freedom of conscience and free profession,
opposed to each other. Sometimes liberty opens new avenues practice and propagation of religion.
of social mobility which eventually helps in bringing more Article 26 - Freedom to manage religious affairs.
equality in the society. Hence, an ideal society is such where Article 27 -- Freedom from payment of taxes for promotion
both liberty and equality complement each other. of any religion.
Statement d is correct: The Constitution of India promotes Article 28 Freedom from attending religious instruction or
harmonious relationship between liberty and equality. worship in certain educational institutions.
In India, the fundamental rights enjoy supremacy over
Important Tips
the Directive Principles. But the Parliament can amend
Fundamental rights available only to citizens and not to
the fundamental rights for implementing the Directive foreigners -
Principles, so long as the amendment does not damage or
Article 15 - Prohibition of discrimination on grounds of
destroy the basic structure of the Constitution. religion, race, caste, sex or place of birth.
5. Solution: (c) Article 16 - Equality of opportunity in matters of public
employment.
Exp) Option c is the correct answer.
Article 19 Protection of six rights related to freedom
Fundamental Rights can be suspended during the operation - (a) of speech and expression; (b) to assemble peaceably
of a National Emergency except the rights guaranteed by and without arms; (c) to form associations or unions; (d) to
Articles 20 and 21. Further, the six rights guaranteed move freely throughout the territory of India; (e) to reside
by Article 19 can be suspended only when emergency is and settle in any part of the territory of India; and (f) to
practice any profession, or to carry on any occupation,
declared on the grounds of war or external aggression
trade or business.
(i.e., external emergency) and not on the ground of armed
Article 29 - Protection of la guage, script and culture
rebellion (i.e., internal emergency). of minorities.
Option a is incorrect. Their application to the members of Article 30 - Right of minorities to establish and administer
armed forces, paramilitary forces, police forces, intelligence educational institutions.
agencies and analogous services can be restricted or
7. Solution: (b)
abrogated by the Parliament (Article 33).
Exp) Option b is the correct answer.
Option b is incorrect. Fundamental Rights are defended
and guaranteed by the Supreme Court. Hence, the aggrieved The phrase 'reasonable restriction connotes that the
person can directly go to the Supreme Court, not necessarily limitation imposed upon a person in enjoyment of the right
should not be arbitrary or of an excessive nature, beyond
by way of appeal against the judgement of the high courts.
what is required in the interest of the public. The restrictions
Option d is incorrect. Most of Fundamental Rights are must be there but they need to be reasonable in nature. Eg.
directly enforceable (self-executory) while a few of them Restrictions mentioned under Article 19(2) considered to be
can be enforced on the basis of a law made for giving effect reasonable restrictions over Article 19.
to them. Such a law can be made only by the Parliament and
Important Tips
not by state legislatures so that uniformity throughout the
country is maintained (Article 35). The freedoms enumerated in Art.19 ( 1) am those great
and basic rights, which are recognized as the natural
6. Solution: (c) rights inherent in the status of a citizen. But none of these
Exp) Option c is the correct answer. freedoms is absolute or uncontrolled, for ench is liable to
be curtailed by laws made or to be made by the state to the
Fundamental rights available to both citizens and foreigners extent mentioned in clauses (2) to (6) of Art 19. Clauses (2)
(except enemy aliens) - to (6) recognize the right of the State to make laws putting
Article 14 - Equality before the law and equal protection of reasonable restrictions in the interests of the general public,
laws. security of the State, public order, decency, or morality and
Article 20 - Protection in respect of conviction for offences. for other reasons set out in those sub-clauses.

Article 21 - Protection of life and personal liberty. 8. Solution: (b)


Article 21A - Right to elementary education. Exp) Option b is the correct answer.
Article 22 - Protection against arrest and detention in Statement 1 is correct. Fundamental Rights act as limitations
certain cases. on the tyranny of the executive and arbitrary laws of the
legislature. In short, they aim at establishing 'a government
Article 23 - Prohibition of traffic in human beings and
of laws and not of men. They prevent the establishment
forced labour. of an authoritarian and despotic rule in the country, and
Article 24 - Prohibition of employment of children in protect the liberties and freedoms of the people against
factories etc. the invasion by the State

111--------------------------------------------Practicebook

KING R QUEEN P [ऋषभ राजपूत ]


II INDIAN POLITY AND GOVERNANCE II
Statement 2 is incorrect. They are not absolute but qualified. segregation of powers between various organs of the state,
The state can impose reasonable restrictions on them. viz., the executive, legislature and the judiciary. The concept
However, whether such restrictions arc reasonable or not of division of powers refer to the segregation of powers
is to be decided by the courts. Thus, they strike a balance between various levels of the government, viz., the centre,
between the rights of the individual and those of the society the states and local governments.
as a whole, between individual liberty and social control. Option c is correct. Article 13 is an example of the system
Statement 3 is correct. Some of them are available only to of checks and balances in India. It related to concept of
the citizens while others are available to all persons whether judicial review where the judiciary can strike down laws
citizens, foreigners or legal persons like corporations or made by the parliament that are deemed ultra-vires to the
companies, e.g., equality before law and equal protection of constitution.
laws (Article 14).
Option d is incorrect. Single party majority in the
9. Solution: (b) parliament weakens the system of checks and balances as
the executive and legislature both are controlled by a single
Exp) Option b is the correct answer.
entity or a single political party.
Statement (a) and (c) are correct: A right is essentially
Knowledge Base: The concept of separation of powers was
an entitlement or a justified claim. It denotes what we are
entitled to as citizens, as individuals and as human beings. given by French philosopher Montesquieu.
In most cases the claimed rights are directed towards the 12. Solution: (d)
state.
Exp) Option d is the correct answer.
Statement (b) is incorrect. Duties are not the claims of
Statement l is correct. In the theory of social justice, the
citizens against the state. But it is the rights which acts as the
proportionate justice principle indicates rewarding people
claims of citizens against the state.
in proportion to the scale and quality of their effort. Thus,
Statement (d) is correct. At the same time, duties are the proportionality provides balance to the principle of equal
moral obligations on the all the citizens towards state. treatment.
They compel us to think not just of our own personal needs
Statement 2 is correct. One of the principles of social justice
and interests but to defend some things as being good for all
is the principle of treating equals equally. It is considered that
of us.
all individuals share certain characteristics as hPman brings.
10. Solution: (c) Therefore, they deserve equal rights and cq\1:.d t1n1ln1ent.
Exp) Option c is the correct answer. The principle of treating equals equally rcqLtire th:,t people
should not be discriminated agai.nst on grounds of class,
Statements l, 2 and 4 are correct. The idea of 'Rule of law'
caste, race or gender. They shonld be judged on the basis of
includes -
their work and acti.ons. Hence, d;,e ,i. .nd equal consideration
Equality before law - Every person, irrespective of any must be given to aH individuals,
differences on the basis of gender, caste, place of birth
and the like, has to be accorded same treatment in the Statement 3 is cm·r.ecL According to the ideal of social
exercise of law. justice, even nondiscrimin,1\i{;n b,:tween people and
rewarding them proportionately h> their efforts might not be
Absence of arbitrary power - Th.is would require that all
enough. The principle of taking a<:count of the special needs
authorities vested with powers of the State are bound by
of people is required. It docs not neces,.adly contradict the
laws themselves in the exercise of their powers,
principle of equal treatment so much as extend it because the
The primacy of the rights of individual - Rule of law principle of treating equals equally could imply that people
has to ensure that individuals' liberty and civil rights are who are not equal in certain important respects could be
protected from the State's and private individual' actions. treated differently.
Statement 3 is incorrect. Rule oflaw requires the government
13. Solution: (a)
which has to be responsible to citizens, not the other way
round. Exp) Option a is the correct answer.
Statement l is incorrect. Creamy layer is a category under
11. Solution: (c)
Other Backward Classes category. They do not get the benefit
Exp) Option c is the correct answer. of the OBC reservation. Creamy layer is the section of people
Option a is incorrect. The system of checks and balances in the Other Backward Classes who are considered socially
enables e ·'.·, of the three branches of government to limit advanced.
the power, of the others. This way, no one branch becomes Statement 2 is incorrect. The 'creamy layer' concept
too powerful. was introduced in the Supreme Court's Indra Sawhney
Option b is incorrect. The system of checks and balances judgment in 1992. The court found it necessary to identify
is antithetical to the system of separation of powers and sections of backward classes who were already "highly
not division of powers. Separation of powers refer to the advanced socially as well as economically and educationally".

Practicebook---------------------------------------------,IJI

KING R QUEEN P [ऋषभ राजपूत ]


;II
II INDIAN POLITY AND GOVERNANCE

This was called creamy layer. The judgment directed the State is not bound to make reservations for SC/ST in the
State governments to identify the 'creamy layer' and exclude matter of promotions.
them from the purview of reservation. The Constitution
It stated that if a State wants to provide reservation to the
does not define the term creamy layer.
SC/ST communities in promotions:
Statement 3 is correct. The Supreme court while quashing
a Haryana government notification of August 17, 2016 It has to collect quantifiable data showing backwardness
held that economic criterion cannot be the sole basis for of the class.
deciding the creamy layer from among the OBCs. Show inadequacy of representation of that class in
14. Solution: (b) public employment in addition to compliance of Article
335.
Exp) Option b is the correct answer.
Statement 1 is correct: Under Article 341, 342 and 342A State needs to ensure that its reservation provision does
the President initially includes various groups/ tribes/ not lead to excessiveness- breaching the ceiling-limit of
castes into the central lists identifying SC, ST and Socially 50%, or destroying the creamy layer principle.
& economically Backward Classes (SEBC) respectively. Any Statement 4 is incorrect: In Jarnail Singh v L.N. Gupta
exclusion or modification thereafter is done by the decision
(2018) case, SC refused to refer the Nagaraj judgment to a
of the Parliament (not the President).
higher bench but later altered the decision by saying that
Statement 2 is incorrect. The Supreme court while ruling
states will not be required to present quantifiable data of
on the validity of Maratha reservation case in Jaishree
backwardness of SC/ST communities.
Laxmanrao Pati v The Chief Minister and others, said that
there is to be only 1 list identifying SEBCs - the central list, Important Tips
which will be modified by the Parliament. Further it said
The term 'Creamy layer' was first mentioned in the
that the states can only make recommendations regarding
Supreme Court's landmark judgment in the Indra Sawhney
inclusions and exclusions. But recently the Parliament passed
case (1992).
the 127th Constitutional Amendment Bill (adding Article
342A(3)), which clarified that the Parliament will only Supreme Court in 2020 ruled that reservation in the matter
modify the Central List and that each state has the power to of promotions in public posts is not a fundamental right,
create and modify its State List regarding identification of and a state cannot be compelled to offer the quota if it
SEBCs in their territory. chooses not to

Statement 3 is correcL The Supreme Court in Jaishree 16. Solution: (b)


Laxmanrao Pati v The Chief Minister and others stated, as
Exp) Option b is the correct answer.
well as according to Artide,; 15 and 16 of the Constitution
•· once identified, each state has the right to take steps for Statement l is incorrect. The concept of 'equality before
advancement and welfare of vulnerable groups like SC, ST law' connotes the absence of any special privileges in favour
and SEBCs. These measures will be applicable within the of any person. The tern, also rneans equal subjection of all
territc-dal limits of that state. There was no confusion or persons to the ordinary law of the land.
debate about the right of states to take affirmative action.
Statement 2 is correct. The concept 'equal protection
Only the power to identi.fy SEBCs was under debate.
of law' connotes the equality of treatment under equal
15. Solution: (c) circumstances, both in the privileges conferred and liabilities
Exp) Option c is the correct answer. imposed by the laws. This also means the similar application
Statement l is correct. The Nagaraj case (2006) in Supreme of the same laws to all persons who are similarly situated.
Court relates to the issue of reservatfons in promotions for
Statement 3 is correct. The term 'equal protection of law'
the SC and ST communities.
also means that the like should be treated alike without any
Statement 2 is correct. The court in M. Nagaraj case had
discrimination.
upheld the application of creamy layer principle to members
of the Scheduled Caste/Scheduled Tribe communities in 17. Solution: (b)
promotions. In this case applying the creamy layer concept
Exp) Option b is the correct answer
in SC/ST reservation in promotions, the Supreme court
reversed its earlier stance in the Indra Sawhney case (1992), The 'equality before law' connotes:
in which it had excluded the creamy layer concept on SCs/ (1) Statement l is correct: The absence of any special
STs (that was applicable on OBCs). privileges in favour of any person.
Statement 3 is correct. The five-judges Bench in Nagaraj
(2) The equal subjection of all persons to the ordinary law of
case upheld the constitutional validity of all 77'\ 81 st , 82 nd ,
the land administered by ordinary law courts.
and 85 th constitutional amendments enabling reservation
of SC/ST communities in promotions, but made certain (3) No person (whether rich or poor, high or low, official or
directives for the states: non-official) is above the law.

IJ:lf---------------------------------------------Practkebook

KING R QUEEN P [ऋषभ राजपूत ]


II INDIAN POLITY AND GOVERNANCE II
The 'equal protection of laws' on the other hand, connotes: "pr<>.srribing" a requirement of residence for jobs in a
(l) The equality of treatment under equal circumstances. particular state. This power vests solely in the Parliament,
not state legislatures.
(2) The similar application of the ,q;inH' laws to all pernon~
who are similarly situated. Hence, statement 2 is In 2002, the Supreme Court invalidated appointment of
incorrect. government teachers in Rajasthan in which the state selection
(3) The like ,!1vu!J be treated alike without any board gave preferenLe tu "ayylicants belonging to the district
discrimination. Thus, the former is a negative concept or the rural areas of the district concerned".
while the latter is a positive concept. In 2019, the Allahabad High Court struck down a
However, bolh of lhem aim at establishing equality of recruitment notification by the UP-Subordinate Service
legal status, opportunity and justice. Hence, statement 3 Selection Commission which prescribed preference for
is correct. wvmcn who are "original residents" of the UP alone.

18. Solution: (a) 21. Solution: (c)


Exp) Option a is the correct answer Exp) Option c is the correct answer.
Statement l is incorrect. Supreme court has held that both Statement l is correct. Article 16 - Right to equal
right to reservation and the right to reservation in promotion opportunity is available only in case of public employment
is not a Fundamental Right. and not private employment. Article 16 provides for equality
Statement 2 is correct. In the famous Manda! Commission of opportunity for all citizens in matters of employment or
aka Indra Sawhney v. Union of India case, the nine-judge appointment to any office under the State. However, the
bench of the Supreme Court recognized the inequality Supreme Court has said that the right tu 1eser valion is not a
among different castes and the held that it would be legal fundamental right.
for the state to categorise backward classes as 'backward' and
Statement 2 is correct. The Right to assemble peacefully
'more backward' for the purpose of providing reservation.
is provided under Article 19. As per the recent Supreme
Statement 3 is incorrect. In the Indra Sawhney vs UOI Court. Judgement, Article 19 can now be enforced even
( 1992) case, the Supreme court held that the backwardness against persons other than State or its instrumentalities. Thus
of a community cannot be determined exclusively with
right to assemble peacefully is protected against inclividual
reference to economic criteria. Backwardness is determined
action as well.
v11 the basis of 'social cond1t10n'. Educational and economic
backwardness can be used as a tool to determine social 22, Solution: (b)
backwardness.
Exp) Option b is the correct answer.
19. Solution: (d) Statement 1 is incorrect. The term 'untouchability' has
Exp) Option dis the correct answer. not been defined either in the Constitution or in the
Right to equality (Articles 14- 18) includes the following: Untouchability (Offences) Act, 1955.

Equality before law and equal protection of laws (Article Statement 2 is correct. In 1976, the Untouchabilily
14). (Offences) Act, 1955 was comprehensively amended and
renamed as the Protection of Civil Rights Act, 1955 to
Prohibition of discrimination on grounds of religion,
race, caste, sex or place of birth (Article 15). enlarge the scope 8nd make penal provisions more stringent.

Equality of opportunity in matters of public employment Statement 3 is correct. The Supreme Court has held that
(Article 16). the right under Article 17 is available against private
individuals and it is the constitutional obligation of the
Abolition of untouchability and prohibition of its
practice (Article 17). State to take necessary action to ensure that this right is not
violated.
Abolition of titles except military and academic (Article
18). 23. Soluliou: ( <l)
20. Solution: (c) Exp) Option d is the correct answt:r,
Exp) Option c is the correct answer, Statement 1 is incorrect. Freedom of speech is a qualified
Article 16 states that "no citizen shall, on grounds only of right. Qualified right is where the state can lawfully interfere
religion, race, caste, sex, descent, place of birth, residence under certain circumstances as laid down in the constitution
or any of them, be ineligible for, or discriminated against or as the need maybe.
in respect or, any employment or office under the State". Statement 2 is incorrect. Abolition of untouchubility is an
The provision is supplemented by the other clauses in the absolute right. Absolute rights arc rights whid, 1..auuol be
Constitution that guarantee equality. limited in any way. An absolute right is a legally enforceable
However, Article 16(3) of the Constitution provides an right; a right to act or refuse to act that is at the sole discretion
exception by saying that Parliament may make a law of the person possessing the right.

Practicebook--------------------------------------------111

KING R QUEEN P [ऋषभ राजपूत ]


II INDIAN POLITY AND GOVERNANCE II
24. Solution: (a) a particular state. However, this power vests solely in the
Exp) Option a is the correct answer. Parliament, and not state legislatures.
Article 18 abolishes titles and makes four provisions in that Statement 2 is correct. Article 16 of the Constitution of
regard: India provides for equality of opportunity for all citizens in
public employment only. Article 16 (1) states that there shall
(a) It prohibits the state from conferring any title (except a
be equality of opportunity for all citizens in matters relating
military or academic distinction) on anybody, whether
to employment or appointment to any office under the State.
a citizen or a foreigner (Statement l is incorrect).
Further, Article 16 (2) says that, no citizen shall, on grounds
(b) It prohibits a citizen of India from accepting any title only of religion, race, caste, sex, descent, place of birth,
from any foreign state. residence or any of them, be ineligible for, or discriminated
(c) A foreigner holding any office of profit or trust under against in respect or, any employment or office under the
the state cannot accept any title from any foreign state State.
without the consent of the president.
27. Solution: (c)
(d) No citizen or foreigner holding any office of profit or
trust under the State is to accept any present, emolument Exp) Option c is the correct answer
or office from or under any foreign State without the Statement a is correct. The Right to form association under
consent of the president. Article 19(1)(c) of the Constitution ofindia not only includes
Statement 2 is correct. In 1996, the Supreme Court upheld the right to start an association or union but also to continue
the constitutional validity of the National Awards-Bharat with the association or union as such. Further, it covers the
Ratna, Padma Vibhushan, Padma Bhushan and Padma Shri. negative right of not to form or join an association or union.
It ruled that these awards do not amount to 'titles' within the Statement b is correct. Reasonable restrictions can be
meaning of Article 18 that prohibits only hereditary titles of imposed on the exercise of this right by the State on the
nobility. grounds of sovereignty and integrity of India, public order
and morality.
25. Solution: (a)
Statement c is incorrect. The right to obtain recognition of
Exp) Option a is the correct answer the association is not a fundamental right. So, it is not part
Statement l is incorrect: In Balaji Raghavan vs Union of of right to form association which is a Fundamental right.
India case, the supreme court of India held that 'the Bharat Statement d is correct. Article 19(1) (c) states that all
Ratna and the Padma awards cannot be categorized under citizens have the right to form associations or unions or
Article 18(1) as they are not titles but merely an honorary cooperative societies. It is applicable to Indian citizens only
award', and not to foreigners.
Statement 2 is incorrect. The Padma Awards Committee
is headed by the Cabinet Secretary and includes Home 28. Solution: (c)
Secretary, Secretary lo the President and four to six eminent Exp) Option c is the correct answer.
persons as members. The recommendations of the committee Option a is incorrect. The freedom to travel abroad has
are submitted to the Prime Minister and the President of been adjudged by the Supreme Court to be a part of Right to
India for approval. Life and Liberty under Article 21 (and not article 19) in the
Statement 3 is correct. The nomination process is open to the Maneka Gandhi v/s Union of India case, 1978.
public. Even self-nomination can be made. The government The right to move freely within the Indian territory is
allows common citizens to submit nominations for Padma protected under Article 19.
Awards. The nominations, when invited, can be submitted Option b is incorrect. The freedom to freely conduct trade
online on the official Padma Awards website, padmaawards. and commerce throughout the territory of India has been
gov.in. protected under Article 301 and not Article 19. Article 301
Statement 4 is incorrect. All persons without distinction says that no state can frame laws/ rules that disadvantage
of race, occupation, position or sex are eligible for these trade and commerce of a trader from 1 part of India with
awards. However, Government servants including those respect to traders of that state.
working with PSUs, except doctors and scientists, are not Option c is correct. Article 19(l)(g) protects the right to
eligible for these Awards practise any profession, or to carry on any occupation, trade
26. Solution: (c) or business.
Exp) Option c is the correct answer. At the same time, the State can make any law relating to,
Statement 1 is correct. The Constitution of India allows 1. the professional or technical qualifications necessary for
prescribing residence as a requirement for jobs in a practising any profession or carrying on any occupation,
particular state. Article 16 (3) of the Constitution of India trade or business, or
provides an exception by saying that Parliament may make 2. the carrying on by the State, or by a corporation owned
a law "prescribing" a requirement of residence for jobs in or controlled by the State, of any trade, business,

1111---------------------------------------------Practicebook

KING R QUEEN P [ऋषभ राजपूत ]


II INDIAN POLITY AND GOVERNANCE II
industry or service, whether to the exclusion, complete The stated purpose of the National Security Act of 1980 is
or partial, of citizens or otherwise "to provide for preventive detention in certain cases and for
Option d is incorrect. The protection of citizens against matters connected therewith".
Forced labour and its forms like Begari has been provided Statement 2 is correct. The amendments to the existing Act
by the constitution under Article 23 (Prohibition of Traffic redefine "Who may commit terrorism ", establishing that
in Human Beings and Forced Labour) and not Article 19. under the Act, the Centre may designate an organization as
a terrorist organization if it commits or participates in acts
29. Solution: (c)
of terrorism, prepares for terrorism, promotes terrorism, or
Exp) Option c is the correct answer. is otherwise involved in terrorism. The Act also additionally
Statement a is correct: The safeguards provided for a person empowers the government to designate individuals as
under JJunilive detention under Article 22 are not available terrorists on the same grounds.
to enemy aliens. For persons under punitive detention, Statement 3 is incorrect. Under UAPA, both Indian and
following rights are conferred - foreign nationals ,811 be charged. The offenders will he
1. Right to be informed ot the grounds of arrest. charged in the same manner whether the act is performed in
2.. Right to consult and be defended by a legal practitioner. a foreign land, outside India.
3. Right to be produced before a magistrate within 24 Statement 4 is incorrect. The Act also paves the way for the
hours, excluding the journey time. National Investigation Agency (NIA) to seize property as part
of investigations into terror cases. Also, while the existing
Statement b is correct: The Parliament has exclusive
Act provides for investigation of cases to be conducted by
authority to make a law of preventive detention for reasons
officers of the rank of Deputy Superintendent or Assistant
connected with defence, foreign affairs and the security of
Commissioner of Police or above. The amended Act now
[ndia. Both the Parliament as well as the state legislatures can
concurrently make a law of preventive detention for reasons additionally empowers the officers of the NIA to investigate
connected with the security of a state, the maintenance of cases - of the rank of Inspector or above.
public order and the maintenance of supplies and services 32. Solution: (b)
essential to the community.
Exp) Option b is the correct answer.
Statement c is incorrect: A person arrested under a
Statement l is correct. Right to Protest is a constitutional
preventive detention law does not have a right to be
right enshrined in Article 19(1) (a), Article 19(1) (b) and
produced before a magistrate within 24 hours. That right
Article 19(1) (c) of the Indian Constitution. Supreme Court
is available only to persons arrested or detained under a
in its recent order in the farm laws protest in Delhi also
punitive detention law.
stated that farmers have a constitutional right to continue
Statement d is correct: As per Article 22, the preventive with their protest.
detention of a person cannot exceed three months unless
Statement 2 is incorrect. Like other fundamental rights,
advisory board reports sufficient cause for extended
right to protest is also not absolute and also subject to
detention. The board is to consist of judges of a high court.
reasonable restrictions mentioned under Article 19(2) and
Knowledge Base: Arrest and detention under first part of 19(3) on the following grounds;
Article 22 do not cover civil arrests.
1. ln the interests of the sovereignty & integrity of India,
30. Solution: (c) 2. The security of the State,
Exp) Option c is the correct answer. 3. Friendly relations with foreign States,
In a significant ruling in January, 2020, the Supreme Court 4. Violation of Public order,
had said that access to the Internet is a fundamental right
5. Decency or morality or in relation to Contempt of
under Article 19 of the Constitution. The bench said that
court, defamation or incitement of an offence
access to Internet is a fundamental right under Article 19
of the Constitution, subject to some restrictions. It said The grounds of restrictions based on Violation of public
that, magistrates, while passing prohibitory orders, should order can be reasonable only when there is evidence that
apply their mind and follow doctrine of proportionality. It protesters will incite lawless or disorderly acts and that such
also clarified that Section 144 CrPC (prohibitory orders) acts are likely to occur.
cannot be used indefinitely to suppress freedom of speech Statement 3 is correct. In the Shaheen Bagh judgement,
and expression and difference of opinion. Supreme Court (SC) upheld the right to peaceful protest
against the law but also cleared that public ways and public
31. Solution: (c)
spaces cannot be occupied and that too indefinitely.
Exp) Option c is the correct answer.
Important Tips
Statement l is incorrect. Unlawful Activities (Prevention)
Act does not contain provision for preventive detention of Article 19 provides the right to assemble peacefully
any individual. without arms limited to reasonable restrictions.

Practicebook----- -----------------lllll
mForu1nm

KING R QUEEN P [ऋषभ राजपूत ]


II INDIAN POLITY AND GOVERNANCE II
33. Solution: (d) spoken or written, or by signs, or by visible representation,
Exp) Option d is the correct answer. or otherwise, brings or attempts to bring into hatred or
contempt, or excites or attempts to excite disaffection
Option a is incorrect. Right to be Forgotten also includes
towards the government established by law in India".
the right to take down a court order. In its order in the
Disaffection includes disloyalty and all feelings of enmity.
Jorawer Singh Mundy case 2021, the Delhi HC also noted
However, comments without exciting or attempting to
that where a court order is demanded to be taken down, the
excite hatred, contempt or disaffection, will not constitute
court will be required to examine the right to privacy of the
an offence under this section.
petitioner on one hand, and the right to information of the
public and maintenance of transparency in judicial records Option c is correct. Sedition is a non-bailable offence under
on the other hand. the Section 124A. Punishment ranges from imprisonment up
to three years to a life term, to which fine may be added.
Option b is incorrect. There is no law in India that at
present recognize and provides for the right to be forgotten. Option d is correct. A person charged under this law is
However, the Personal Data Protection Bill 2019 recognised barred from a government job. They have to live without
their passport and must produce themselves in the court at
this right. Section 20 of the Bill gave an individual the right
all times as and when required.
to restrict or prevent the continuing disclosure of their
personal data when such data ( 1) has served the purpose 36. Solution: (c)
for which it was collected, or is no longer necessary for said
Exp) Option c is the correct answer
purpose; (2) was made with the conseut of individual, which
consent has since been withdrawn; or (3) was made contrary Statement l is correct. Recently, the Supreme court said
to the PDP Bill or any law in force. However, the provision that the right to health is a fundamental right guaranteed
made this right enforceable only on an order passed by the under Article 21 of the Constitution. Right to health
adjudicating officer appointed under the Bill. includes affordable treatment. The Bench said the State and
local authorities should either make more provisions for
Option c is incorrect. The right to be forgotten has been
affordable treatment in their hospitals or there shall be cap
recognised as a statutory right in the European Union on the fees charged by the private hospitals in exercise of the
(not USA) under the General Data Protection Regulation powers under the Disaster Management Act
(GDPR), and has been upheld by a number of courts in the
Statement 2 is correct. The imposition of president's rule
United Kingdom, and in Europe.
or State emergency does not affect citizen's Fundamental
Option d is correct. The right to be forgotten and the right rights.
to be left alone are "inherent aspects" of the right to privacy
as per a Delhi HC judgement of 2019. 37. Solution: (b)

Important Tips Exp) Option b is the correct answer

The 'right to be forgotten' is the right to have publicly The Supreme court has held that speedy trial is an inalienable
available personal information removed from the internet, right under Article 21 of the constitution and hence no
search, databases, websites or any other public platforms, person shall be deprived of his life and liberty without the
once the personal information in question is no longer procedure of law and the procedure of law must be 'fair',
necessary, or relevant. 'reasonable', and 'just'.
--------~--------~
Article 21 comes under the category of the right to freedom.
34. Solution: (b)
Exp) Option b is the correct answer. 38. Solution: (a)

The Supreme Court has held that failure on the part of Exp) Option a is the correct answer.
government hospitals to provide timely medical treatment Statement a is incorrect. Article 22 (2) provides that: Every
to a person in need of treatment results in violation of person who is arrested and detained in custody shall be
fundamental right guaranteed under Article 21 (right to produced before the nearest magistrate within a period of
life). It is the obligation of the state to ensure the creation of twenty-four hours of such arrest. However, this provision
congenial conditions for good health. is applicable for punitive detention and not preventive
detention.
35. Solution: (a)
Statement bis correct. Article 22 authorizes the Parliament
Exp) Option a is the correct answer to prescribe the maximum period for which a person can
Option a is incorrect. Sedition law was originally drafted be detained in any classes of cases under a preventive
in 1837 by Thomas Macaulay, but was inexplicably omitted detention law.
when the Indian Penal Code (IPC) was enacted in 1860. It Statement c is correct. The Parliament as well as the state
was later added in 1870 under Section 124A by an amendment legislatures have authority to make a law on preventive
introduced by Sir James Stephen. detention for reasons connected with the security of a state,
Option b is correct. Section 124A of the IPC defines sedition the maintenance of public order and the maintenance of
as an offence committed when "any person by words, either supplies and services essential to the community.

ml----------- --------Practice book

KING R QUEEN P [ऋषभ राजपूत ]


H INDIAN POLITY AND GOVERNANCE II
Statement d is correct. Protection under a preventive The fundamental right to freedom of conscience is protected
detention law, under Article 22, is available to both citizens under Article 25 of the Constitution. Freedom of conscience
as well as foreigners. lt states that the grounds of detention is inner freedom of an individual to mould his relation with
shoL1ld be communicated to the cletenu. However, the facts God or Creatures in whatever way he desires. Article 25 also
considered to be against the public interest need not be protects an individual's right to freely profess, practice and
disclosed. The cletenu should also be afforded an opportunity propagate religion.
to make representation against the detention order. The Supreme Court in its various judgements has declared
Right to privacy, right to health and right to speedy trial as
39. Solution: (cl)
part of Article 21.
Exp) Option dis the correct answer
42. Solution: (a)
Statement l is incorrect. This act empowers the Central
Government and State Governments to detain a person to Exp) Option a is the correct answer
prevent him/her from acting in any manner prejudicial to the Statement l is correct. The supreme court has held that the
security ofindia, the relations oflndia with foreign countries, freedom of speech and expression and the freedom to practice
the maintenance of public order, or the maintenance of any profession or carry on any trade, business or occupation
supplies and services essential to the community. over the medium of internet enjoys constitutional protection
under article 19 (1).
Statement 2 is incorrect. Under the act, the maximum
period of detention is 12 months. Statement 2 is incorrect. Kerala had become the first state
in 2017 to declare access to Internet "a basic human right".
The order can be made by the District Magistrate or a
Commissioner of Police under their respective jurisdictions, 43. Solution: (c)
but the detention should be reported to the State Government Exp) Option c is the correct answer.
along with the grounds on which the order has been made.
After the Indian Independence, when the Constitution of
No such order shall remain in force for more than twelve India came into force on 26th January, 1950, the right to
clays unless approved by the State Government. property was included as a 'fundamental right' under Article
Statement 3 is incorrect. The NSA, 1980 provides that 'the 19(l)(f) and Article 31 in Part III, making it an enforceable
Central Government or the State Government can prevent a right.
person from acting in any manner prejudicial to the defence Parliament passed the Constitution 44th Amendment
of India, the relations of India with foreign powers, or which made right to property an ordinary legal right and
the security of India, and for this purpose the appropriate constitutional right under Article 300-A.
authority can detain such person.'
44. Solution: (c)
The act explicitly mentions that 'person' includes foreigner.
Exp) Option c is the correct answer
40. Solution: (cl) The Right against Exploitation is enshrined in Articles 23
Exp) Option cl is the correct answer. and 24 of the Indian Constitution.
Statement l is incorrect. Section 144 CrPC empowers a district Statement l is correct. Article 23 prohibits traffic in
magistrate to issue orders to prevent and address urgent cases human beings, begar (forced labour) and other similar
of apprehended danger or nuisance. An order passed under forms of forced labour.
this can remain in force for more than one month. However, Statement 2 is incorrect. Abolition of untouchability and
any order passed under Section 144 cannot remain in force prohibition of its practice is provided under the Right to
for more than two months from the date of the order, unless Equality in Article 17.
the state government considers it necessary. Even then, the Statement 3 is incorrect. Cultural and Educational Rights
total period cannot extend to more than six months. (Article 29 and 30):
Statement 2 is incorrect. The Supreme Court has upheld the Article 29 provides for the protection of language, script
constitutionality of Section 144 of the CrPC. However, the and culture of minorities.
court held that the restrictions under sec 144 of the CrPC Article 30 provides for the right of minorities to
cannot be used to suppress legitimate expression and are establish and administer educational Institutions.
subject to judicial scrutiny. Statement 4 is correct. Article 24 prohibits the employment
Statement 3 is incorrect. The Temporary Suspension of of children below the age of 14 years in any factory, mine or
Telecom Services (Public Emergency or Public Service) Rules other hazardous activities like construction work or railway.
under the Telegraph Act, 1985 governs the suspension of But it does not prohibit their employment in any harmless or
Internet. According to these Rules only the Home Secretary innocent work.
of the Union or a state can pass an order to shut down the
45. Solution: (a)
internet. The order must include the reasons for decision.
Exp) Option a is the correct answer.
41. Solution: (c) Statement l is incorrect. As per Section 43D(5) of the
Exp) Option c is the correct answer. Unlawful Activities (Prevention) Act,1967 bail cannot be

Practicebook--------------------------------------------111

KING R QUEEN P [ऋषभ राजपूत ]


II INDIAN POLITY AND GOVERNANCE II
granted to a suspect if the court is of the opinion that there of any particular religion or religious denomination. But it
are reasonable grounds to believe that the charges are prima does not prohibit levying of a fee as the purpose of a fee is
facie true. for the secular administration of a religious denomination.
Statement 2 is correct. Due to section 43D(5), bail becomes Statement 2 is correct: Article 26 states that every religious
extremely difficult to get under Unlawful Activities denomination has the right to own and acquire movable and
(Prevention) Act, 1967. Also, a supreme court judgement immovable property.
in Watali case has clarified that the court considering bail Statement 3 is incorrect: Article 25 gives all persons the
should not examine the evidence too deeply, but must go by freedom of conscience and the right to freely profess, practice
the prosecution version based on broad probabilities. This and propagate religion. All persons include foreigners as
means that the onus is on the accused to show that the well.
case is false but without inviting the court to evaluate the
available evidence. 48. Solution: (a)
Statement 3 is incorrect. Parliament has exclusive authority Exp) Option a is the correct answer.
to make laws on preventive detention for reasons connected Statement 1 is correct. Implying from Article 27, the State
with defence, foreign affairs and the security of India. The should not spend the public money collected by way of tax
Constitution has divided the legislative power with regard for the promotion or maintenance of any particular religion.
to preventive detention between the Parliament and state This provision prohibits the State from favouring, patronising
Legislatures. and supporting one religion over the other. This means that
Important Tips the taxes can be used for the promotion or maintenance of
all religions.
Under Unlawful Activities (Prevention) Act,1967 both
Indian and foreign nationals can be charged. It will be Statement 2 is incorrect. Under Article 28, no religious
applicable to the offenders in the same manner, even if instruction shall be provided in any educational institution
crime is committed on a foreign land, outside India. wholly maintained out of State funds. Also, religious
instructions are not completely prohibited in institutions
46. Solution: (c) receiving aid from the State. They are permitted on a
Exp) Option c is the correct answer. voluntary basis. A person's consent is needed in such cases.
Statement l is correct. A religious denomination is a sub- 49. Solution: (d)
group within a larger religious organisation that follows a
Exp) Option d is the correct answer.
common faith, practice, has an identity and traditions.
Article 25 says that all persons are equally entitled to freedom
Article 26 talks about religious denomination and their
of conscience and the right to freely profess, practice and
rights.
propagate religion.
The Supreme Court held that a religious denomination must
Option a is incorrect. Right to propagate under Article 25
satisfy three conditions:
does not include right to convert another person to one's
(a) It should be a collection of individuals who have a system own religion. Forcible conversion impinge on the 'freedom
of beliefs (doctrines) which they regard as conducive to of conscience' available to all the persons alike.
their spiritual well-being;
Option b is incorrect. Article 25 covers not only religious
(b) It should have a common organisation; and beliefs (doctrines) but also religious practices (rituals).
(c) It should be designated by a distinctive name. Moreover, these rights are available to all persons-citizens
Statement 2 is incorrect. The supreme court has held that as well as non-citizens.
"Devotees of Ayyappa do not constitute a separate religious Option c is incorrect. According to Article 26 and not Art
denomination.". The 5 judges bench observed that barring 25, every religious denomination or any of its section shall
the entry of women, when they are in the menstruation have right to establish and maintain institutions for religious
age, to the Sabarimala temple was not an essential part of and charitable purposes. Article 25 guarantees rights of
the religion. Denying Hindu women entry into the temple individuals while Article 26 protects collective freedom of
violated their fundamental right to practice religion. religion.
Statement 3 is correct. In the legal framework, the doctrine Option d is correct. Article 25 covers not only religious
of essentiality is a doctrine that has evolved to protect the beliefs (doctrines) but also religious practices (rituals).
religious practices that are essential or integral and does not
Important Tips
violate any fundamental right.
The rights under Art 25 are subject to public order,
47. Solution: (a) morality, health and other provisions relating to
fundamental rights.
Exp) Option a is the correct answer.
Statement l is incorrect: Article 27 lays that no person shall 50. Solution: (c)
be compelled to pay any taxes for promotion or maintenance Exp) Option c is the correct answer.

mJlf---------------------------------------------Pradicebook
i1Foru111WA'I

KING R QUEEN P [ऋषभ राजपूत ]


II INDIAN POLITY AND GOVERNANCE II
Statement l is incorrect. Presently the linguistic minorities Article 32 and not any other right like non-fundamental
are identified on a state-wise basis thus determined by constitutional rights, statutory rights, and customary
the state government whereas religious minorities are rights and so on. In other words, the Supreme Court, under
determined by the Central Government. Article 32, cannot determine a question that does not
Statement 2 is correct. Under Article 29, any section of involve Fundamental Rights. Article 32 cannot be invoked
the citizens residing in any part of India having a distinct simply to determine the constitutionality of an executive
language, script or culture of its own, shall have the right to order or a legislation unless it directly infringes any of the
conserve the same. Supreme court held that the scope of this fundamental rights.
article is not restricted to minorities alone. This is because of Statement 2 is correct. Parliament can empower any other
the use of the word 'section of citizens' in the article includes court to issue directions, orders and writs of all kinds.
minorities as well as majority. However, this can be done without prejudice to the above
Statement 3 is correct. The term minority is not defined powers conferred on the Supreme Court. Any other court
in the Constitution of India. But National commission for here does not include high courts because Article 226 has
minorities act, 1992 defines a minority as "a community already conferred these powers on the high courts.
notified as such by the Ct:ulrnl government." Government Statement 3 is incorrect. In case of the enforcement of
of India has declared six religions namely, Muslims, Fundamental Rights, the jurisdiction of the Supreme Court
Christians, Sikhs, Buddhist and Parsis (Zoroastrian) and Jain is original but not exclusive. It is concurrent with the
as religious minorities in India. jurisdiction of the high court under Article 226. It vests
51. Solution: (a) original powers in the high court to issue directions, orders
and writs of all kinds for the enforcement of the Fundamental
Exp) Option a is the correct answer.
Rights. It means when the Fundamental Rights of a citizen
Statement l is correct. Linguistic minorities have not been are violated, the aggrieved party has the option of moving
defined in the Constitution or by any statute. They get either the high court or the Supreme Court directly.
defined when their mother tongue differs from regional
language of the area. 55. Solution: (a)
Statement 2 is incorrect. The Constitution of India (Article Exp) Option a is the correct answer
350 A) provides that every state must provide primary Statement 1 is correct. Habeas Corpus is a Latin term which
education and not secondary education in mother tongue to literally means 'to have the body of'. It is an order issued by
children of linguistic minorities. the court to a person who has detained another person,
Statement 3 is incorrect. Status of linguistic minorities are to produce the body of the latter before it. The court then
identified by the state government /Union Territory on the examines the cause and legality of detention. It would set the
basis of Census data. detained person free, if the detention is found to be illegal.
52. Solution: (a) Statement 2 is correct. The writ of habeas corpus can be
issued against both public authorities as well as private
Exp) Option a is the correct answer.
individuals.
Statement l is correct. Article 29 grants protection to both
religious minorities as well as linguistic minorities. Statement 3 is correct. The writ is not issued where the:

Statement 2 is incorrect. Article 30 grants all minorities detention is lawful


(religious and linguistic) the right to establish and the proceeding is for contempt of a legislature or a court
administer educational institutions of their choice. But these detention is by a competent court
institutions are not free from all government regulations.
detention is outside the jurisdiction of the court
The general laws of the land relating to national interest,
national security, social welfare, public order, morality, Statement 4 is incorrect. Supreme Court and High Courts
health, sanitation, taxation etc., applicable to all. can issue the writs of habeas corpus, mandamus, prohibition,
certiorari and quo-warranto. Only in the case of Quo-
53. Solution: (b) warranto, it can be sought by any interested party and not
Exp) Option b is the correct answer necessarily by the aggrieved person. In case of Habeas
The right to property was deleted from the list of Corpus, it can be sought by aggrieved person or his family
Fundamental Rights by the 44th Amendment Act, 1978. It or his friends and not by any interested party.
is made a legal right under Article 300-A in Part XII of the
56. Solution: (a)
Constitution.
Exp) Option a is the correct answer
54. Solution: (b)
The supreme court, and High courts have power to issue
Exp) Option b is the correct answer. writs in the nature of habeas corpus, quo warranto ,
Statement l is correct. Only the Fundamental Rights mandamus , certiorari ,prohibition etc., under Arts. 32 and
guaranteed by the Constitution can be enforced under 226 respectively.

--El

KING R QUEEN P [ऋषभ राजपूत ]


II INDIAN POLITY AND GOVERNANCE II
Statement l is correct. The Supreme Court can issue that there is uniformity throughout India with regard to the
writs only for the enforcement of the Fundamental Rights nature of those fundamental rights and punishment for their
and not for other purposes. The high court, on the other infringement.
hand, can issue writs not only for the enforcement of the Statement 2 is incorrect. Article 35 also extends the
fundamental rights but also for other purposes. competence of the Parliament to make a law on the matters
Statement 2 is incorrect. The concept of writs has been of Fundamental Rights, even though some of those matters
borrowed from English Law, where these laws were known may fall within the sphere of the state legislatures (i.e., State
as 'Prerogative writs. List).

57. Solution: (a) 60. Solution: (b)


Exp) Option a is the correct answer Exp) Option b is the correct answer
Statement l is incorrect. Article 35 lays down that the Fundamental Rights can be suspended during the
power to make laws, to give effect to certain specified operation of a National Emergency except the rights
fundamental rights shall vest only in the Parliament and guaranteed by Articles 20 and 21.
not in the state legislatures. This provision ensures that
The 44th Amendment Act of 1978 restricted the scope
there is uniformity throughout India with regard to the
of Article 359 in two ways. Firstly, the President cannot
nature of those fundamental rights and punishment for their
suspend the right to move the Court for the enforcement
infringement.
of fundamental rights guaranteed by Articles 20 to 21.
Statement 2 is correct. A person can move the High Court Secondly, only those laws or associated executive action
by an ordinary suit or under Article 226 (writ jurisdiction
which are related with the emergency are protected from
of high court) in case of violation of rights Outside Part
being challenged in a court of law.
III of the constitution.
Statement 3 is incorrect. Article 24 prohibits the 61. Solution: (d)
employment of children below the age of 14 years in any Exp) Option d is the correct answer.
factory, mine or other hazardous activities like construction
The natural justice mainly focuses on the following:
work or railway. But it does not prohibit their employment
in any harmless or innocent work. 1) An opportunity for fair hearing
2) One cannot be a judge in his own cause
58. Solution: (b)
3) The decision-making authority must act unbiased
Exp) Option b is the correct answer.
Statement 1 is correct. There is no specific or express Important Tips
provision in the Constitution that authorizes the executive to Although the Constitution of India does not mention the
declare martial law. It is implicit in Article 34 under which term 'Natural Justice' anywhere, its principles are applied
martial law can be declared in any area within the territory in various parts:
of India. a. According to Article 14 any violation of natural justice
Statement 2 is correct. Article 34 empowers the Parliament is equivalent to violation of equality.
to indemnify any government servant or any other person b. The introduction of due process by the Supreme Court
for any act done by him in connection with the maintenance under the Article 21 of the Constitution has brought
or restoration of order in any area where martial law was in the principles of natural justice into Article 21.
force. The Act of Indemnity made by the Parliament cannot
be challenged in any court on the ground of contravention 62. Solution: (c)
of any of the fundamental rights. Exp) Option c is the correct answer
Statement 3 is incorrect. Martial law affects only Statement l is correct: Fundamental Rights are not absolute
Fundamental Rights. Unlike national emergency, it does not but qualified. The state can impose reasonable restrictions
affect Centre-state relations, distribution of revenues and on them. However, whether such restrictions are reasonable
legislative powers between centre and states, and the tenure or not is to be decided by the courts. Thus, they strike a
of the Parliament. balance between the rights of the individual and those of
59. Solution: (a) the society as a whole, between individual liberty and social
control.
Exp) Option a is the correct answer.
Statement 2 is correct: Fundamental Rights are defended
Statement l is correct. Article 35 of the Constitution states
and guaranteed by the Supreme Court. Hence, the aggrieved
that the legislature of a state shall not have powers to make
laws for prescribing punishment for those acts that are person can directly go to the Supreme court, not necessarily
declared to be offences under the fundamental rights. These by way of appeal against the judgment of the high courts.
include untouchability (Article 17) and traffic in human Statement 3 is correct: Most of the fundamental rights are
beings and forced labour (Article 23). This provision ensures directly enforceable (self-executory) while a few of them

lmlf----------------------------------------------Practicebook

KING R QUEEN P [ऋषभ राजपूत ]


II INDIAN POLITY AND GOVERNANCE II
can be enforced on the basis of a law made for giving effect 65. Solution: (b)
to them. Such a law can be made only by the Parliament and Exp) Option b is the correct answelt'
not state so that uniformity throughout the
Statement l is correct: Reservation for Scheduled Castes,
country is maintained (Article 35).
Scheduled Tribes, and Other Backward Classes is referred to
Statement 4 is incorrect; Their application to the members of as vertical reservation. It applies separately for each of the
armed forces, para-military forces, police forces, intelligence groups specified under the law.
agencies and analogous services can be restricted or
Statement 2 is correct: Horizontal reservation refers to
abrogated by the Parliament (Article 33).
the equal opportunity provided to other categories of
63. Solution: (d) beneficiaries such as women, veterans, the transgender
Exp) Option d is the correct answer community, and individualo with disabilities. cuttiHg
Article 15 provides that state shall not discriminate against through the vertical categories.
any citizen on grounds only of religion, race, caste, sex Statement 3 is incorrect: Article 1G(4) of the Constitution
or place of birth. The use of word 'only' here means that contemplates vertical reservation while Article 15 (3) of the
discrimination on other grounds is not prohibited. Constitution contemplates horizontal reservation.
However, when these conditions are accompanied by some Article 15( 4) and 16(4) of the Constitution provides the State
other conditions then state can discriminate. For example, and Central Governments to reserve seats in government
only on basis of caste, state cannot provide discriminatory
services for the members of the SC and ST.
reservation policy but caste backwardness can become
Article 15(3) makes it possible for the state to create
eligible ground for the enactment of discriminatory
special provisions for protecting the interests of women and
legislation by the state.
The state is permitted to make any special provision for the children.
advancement of any socially and educationally backward Important Tips
classes of citizens or for the scheduled castes and scheduled
The Supreme Court had ruled that if a person belonging
tribes. to an intersection of vertical-horizontal reserved category
64. Solution: (b) had secured scores high enough to qualify without the
Exp) Option b is the correct answer vertical reservation, the person would be counted as
Option a is incorrect: In 1995, the Supreme Court, in qualifying without the vertical reservation, and cannot be
excluded from the horizontal quota in the general category.
Dalwant Singh v State of Punjab, held that mere sloganeering
which evoked no public response did not amount to sedition. 66. Solution: (c)
Option bis correct: In the landmark 1962 Kedar Nath Singh
Exp) Option e is the correct answer.
case, the Supreme Court upheld the constitutional validity of
the sedition law, it attempted to restrict its scope for misuse. Statement l is incorrect: Right to equality under Article
The court held that unless accompanied by an incitement 14 says that the state shall not deny to any person equality
or call for violence, criticism of the government cannot be before law or the equal protection of the laws within the
labelled sedition. Further, seven principles in the Kedar territory of India. Thus, this right is available only against
Nath Singh ruling specify situations in which the charge of the state but not the private individuals.
sedition cannot Le applied. Statement 2 is correct: The supreme court held that the
Option c is incorrect: Verdict in Kesavananda Bharati case right under Article 17 is available against private individuals
was delivered on 24th April 1973 by a thin majority of 7:6 also and it is the constitutional obligation of the state to take
wherein the majority held that any provision of the Indian necessary action to ensure that this right is not violated.
Constitution can be amended by the Parliament, provided
Statement 3 is correct: Article 23 prohibits traffic in human
that such amendment did not change the Constitution's basic
beings, beggar (forced labour) and other similar forms of
structure. Thus, concept of 'basic structure' came because of
forced labour. It protects the individual not only against the
this verdict.
state but also against private persons.
Option dis incorrect: In I.R. Coelho v. State of Tamil Nadu,
a nine-member bench of Supreme Court held that ninth Statement 4 is correct: Article 15 prohibits the state and
schedule items are not immune to judicial review as it is part individuals both from discriminating any citizen only on the
of the constitution. ground of religion, race, sex, place of birth or any of them
in having access to public places such as parks, clubs, wells,
Important Tips
tanks or roads maintained out of the state's funds.
Sedition, which falls under Section 124A of the Indian
Penal Code, is defined as any action that brings or attempts Important Tips
to bring hatred or contempt towards the government of Article 15, 1'7, 23 and 24 are available against private
India. This was inserted into the Section 124A of IPC in individuals/ corporations and state while rest other rights
1870, by the British. are available against only state.

Practicebook---------------------------------------------1111

KING R QUEEN P [ऋषभ राजपूत ]


II INDIAN POLITY AND GOVERNANCE II
67. Solution: (a) it should be designated by a distinctive name.
Exp) Option a is the correct answer. All three must be fulfilled.
Statement 2 is correct: There are only 3 'reasonable
Statement l is correct: Under Article 17, Untouchability" is
restrictions' to the collective religious freedom rights
abolished and its practice in any form is forbidden.
provided under Article 26, as mentioned in the constitution:
Statement 2 is incorrect: The term 'untouchability' has not
public order
been defined either in the constitution nor in the protection
morality (scope vis a vis Article 25 currently under
civil rights act, 1955. However, Supreme court has defined
consideration)
it as a social practice among Hindu's when a person
solely on the basis of his/her birth looked down upon and health
discriminated wrongly. Other provisions related to fundamental rights are a
restriction on rights guaranteed under Article 25, not
Statement 3 is incorrect: The constitution does not provide
Article 26.
the prescription of the punishment rather the Protection of
Statement 3 is correct: The freedom to manage religious
Civil rights act, 1955 prescribes punishment for the practice
affairs is provided by Article 26. This Article gives the right
of 'untouchability'.
to every religious denomination, or any section thereof, to
Important Tips exercise the rights that it stipulates. However, this right has
Representation of people's act 1951 also prescribes practice to be exercised in a manner that is in conformity with public
of 'untouchability' as a ground for the disqualification order, morality, and health. Clause (a) of Article 26 gives a
of a person from the election to the parliament or state religious denomination the right to establish and maintain
legislature. institutions for religious and charitabie purposes.

68. Solution: (d) 70. Solution: (c)

Exp) Option dis the correct answer Exp) Option c is the correct answer

Statement a is incorrect: Article 19(2) imposes reasonable Statement l is correct. Article 33 empowers the Parliament
restrictions on the basis of sovereignty and integrity of to restrict or abrogate the fundamental rights of the
India, the security of the State, friendly relations with foreign members of armed forces, para-military forces, police forces,
intelligence agencies and analogous forces. The expression
States, public order, decency or morality or in relation to
'members of the armed forces' also covers such employees of
contempt of court, defamation or incitement to an offence.
the armed forces as barbers, carpenters, mechanics, cooks,
Statement b is incorrect: In determining the reasonableness chowkidars, bootmakers, tailors who are non-combatants.
of statute, the court should see both to the nature of the
Statement 2 is correct. The power to make laws under
restriction and procedure prescribed by the statute for
Article 33 is conferred only on
enforcing the restrictions on the individual freedom. Not
only substantive but also procedural provisions of a statute Parliament and not on state legislatures. Accordingly,
also enter into the verdict of its reasonableness. the Parliament has enacted the Act (1950), the Navy
Act (1950), the Air Force Act (1950), the Police Forces
Statement c is incorrect: The reasonableness of a restriction
(Restriction of Rights) Act, 1966, the Border Security Force
has to be determined in an objective manner and from the
Act and so on.
standpoint of the interests of the general public and not from
the point of view of persons upon whom the restrictions are Statement 3 is correct. Any law made by Parliament under
imposed or upon abstract considerations. Article 33 cannot be challenged in any court on the
ground of contravention of any of the fundamental rights.
Statement dis correct: Every citizen has the right to reside
A parliamentary law can also exclude the court martial
and settle in any part of the territory of the country. The
from the writ jurisdiction of the Supreme Court and the high
State can impose reasonable restrictions on the exercise of
courts, so far as the enforcement of Fundamental Rights is
freedom of residence on two grounds- (a) the interests of
the general public and (b) the protection of interests of any concerned.
scheduled tribes. 71. Solution: (b)
69. Solution: (b) Exp) Option b is the correct answer.
Exp) Option b is the correct answer. Overweight male cabin crew are allowed to get promotion
Statement l is incorrect: For a group to be termed as a in the national airlines but their women colleagues who gain
'Religious Denomination' for the purposes of Article 26 weight are penalised.
of the Constitution, following three conditions have to be In the above situation, the Fundamental Right under Article
satisfied: 16 of the Constitution oflndia is violated. Article 16 provides
it should be a collection of individuals with common set for equality of opportunity for all citizens in matters of
of spiritual/ religious beliefs (doctrines) employment or appointment to any office under the State.
it should have a common organisational structure No citizen can be discriminated against or be ineligible for

mlr--------------------------------------------Practicebook

KING R QUEEN P [ऋषभ राजपूत ]


II INDIAN POLITY AND GOVERNANCE II
any employment or office under the State on grounds of Important Tips
only religion, race, caste, sex, descent, place of birth or The ICESCR aims to ensure the protection of economic,
residence. In this case the person is being discriminated social and cultural rights. It aims to ensure:
based on her sex only. the right to self-determination of all peoples (article l);
the right to non-discrimination based on race, colour,
72. Solution: (c) sex, language, religion, political or other opinion,
Exp) Option c is the correct answer national or social origin, property, birth or other status
(article2);
Statement 1 is correct: - Article 29 in the Constitution of
the equal right of men and women to enjoy the rights
India provides for the Protection of interests of minorities. in the ICESCR (article 3);
Article 29 provides that any section of the citizens residing the right to work (articles 6-7);
in any part of India having a distinct language, script or the right to form and join trade unions (article 8);
culture of its own, shall have the right to conserve the same. the right to social security (article 9);
Statement 2 is correct: - No citizen shall be denied admission protection and assistance to the family (article 10);
into any educational institution maintained by the State or the right to an adequate standard of living (article 11);
receiving aid out of State funds on grounds only of religion, the right to health (article 12);
race, caste, or language (Article 29). the right to education (articles 13-14);
Statement 3 is correct: - Cultural and Educational Rights and the right to cultural freedoms (article 15)
safeguards the rights of linguistic, cultural, and religious 75. Solution: (a)
minorities. Articles 29 and 30 deals with the cultural and
Exp) Option a is the correct answer.
educational rights of Indian citizens. Fundamental Rights
The following Fundamental rights are being violated in the
under Article 15, 16, 19, 21, Article 29 and Article 30 are
above instance -
available only to Citizens of India and not to foreigners.
Article 17 within Right to Equality abolish any form of
73. Solution: (d) Untouchability, so in this case discouraging children of caste
groups associated with scavenging from pursuing education
Exp) Option d is the correct answer
is a form of untouchability.
Right to Constitutional Remedies (Article 32) provides
Article 19 (f) within Right to Freedom i.e., it provides
the right to remedies for the enforcement of fundamental
for the right to practice any profession or to carry on any
rights. This makes the fundamental rights real. A mere occupation, but in this case, the people were being forced to
declaration of fundamental rights in the Constitution is their former job and they were denied other jobs which is
meaningless, useless and worthless without provisions of their right to practice any profession they want.
enforcement, in case of violation. That is why Dr. Ambedkar Article 24 within Right to Exploitation also prohibits
called Article 32 as the most important article of the the employment of children below 14 years of age in
Constitution- 'an Article without which this constitution hazardous factories. In this case nothing is being done to
would be a nullity. It is the very soul of the Constitution prohibit the employment of children in these factories and
and the very heart of it'. by discouraging them to pursue education, these children
have no option to work in these hazardous activities of
74. Solution: (b) scavenging. So, this right is also being violated.
Exp) Option b is the correct answer Not Article 30 as it is related to minorities, but Article
Statement l is incorrect. The International Covenant 29 of Cultural and Educational right is being violated as
on Economic, Social and Cultural Rights (ICESCR) is a according to this right no children can be denied admission
multilateral treaty adopted by the United Nations General into any educational institution maintained by the State or
receiving aid out of State funds.
Assembly (UNGA) on 16 December 1966 and it came into
force from 3 January 1976. The Indian Constitution does not recognize property
right as a Fundamental Right. In the year 1977, with the
Statement 2 is correct. India ratified the treaty on 10 April
enactment of the 44th amendment the right to acquire, hold,
1979.
and dispose of property as a Fundamental Right. However, in
Statement 3 is correct. ICESCR (and its Optional Protocol), another part of constitution, Article 300(A) was inserted to
Universal Declaration of Human Rights, and International affirm that no person shall be deprived of his property save
Covenant on Civil and Political Rights (ICCPR) are part of by authority of law. The result is that the right to property as
International Bill of Human Rights. a fundamental right is now substituted as a statutory right.

Practicebook---------------------------------------------,DJ

KING R QUEEN P [ऋषभ राजपूत ]


II INDIAN POLITYAND GOVERNANCE II
76. Solution: (c) endowment. However, it levies fee, as levying a fee is not
Exp) Option c is the correct answer prohibited. Fee is used to control the secular administration
and not to promote any religion.
Statement a is incorrect: - An individual is provided a
number of rights. However, not all of them could be called Statement 2 is correct. Under fundamental right of right to
as Fundamental Rights. The Constitution listed some of equality, Article 16 clearly guarantees that the Government
the very essential rights that would be specially protected will not discriminate on the basis of religion in giving
and called them 'Fundamental Rights. There may be other employment .
ordinary legal rights e.g. right to property, right to vote. Statement 3 is incorrect. Freedom of religion is subject to
Statement b is incorrect: - Fundamental Rights are different certain limitations. The government can impose restrictions
from other rights available to us. While ordinary legal on the practice of freedom of religion in order to protect public
rights are protected and enforced by ordinary law. However, order, morality and health. This means that the freedom of
fundamental rights are protected and guaranteed by the religion is not an unlimited right. The government can
constitution. interfere in religious matters for rooting out certain social
Statement c correct: - The word fundamental suggests evils. For example, in the past, the government has taken
that these rights are so important that the Constitution has steps banning practices like sati, bigamy or human sacrifice.
separately listed them and made special provisions for their
79. Solution: (d)
protection. The Fundamental Rights are so important that
the Constitution itself ensures that they are not violated by Exp) Option d is the correct answer.
the government. Fundamental Rights are protected and Liberty is the state of being free within society from
guaranteed by the constitution of the country. oppressive restrictions imposed by authority on one's
Statement d is incorrect: - Fundamental Rights are behaviour or political views.
not absolute but qualified or subjected to reasonable Statement l is incorrect. Negative liberty (and not positive
restrictions. They can be suspended during the operation liberty) is 'freedom from' constraints. Negative liberty is
of a National Emergency except the rights guaranteed by the absence of obstacles, barriers or constraints. One has
Articles 20 and 21. They are not sacrosanct or permanent. negative liberty to the extent that actions are available to one
The Parliament can curtail or repeal them but only by a in this negative sense.
constitutional amendment act and not by an ordinary act.
'Negative liberty' seeks to define and defend an area in which
Moreover, this can be done without affecting the 'basic
the individual would be inviolable. This is an area in which
structure' of the Constitution. However, whether such
no external authority can interfere. It is a minimum area that
restrictions are reasonable or not is to be decided by the
is sacred and in which whatever the individual does, is not
courts. Thus, they strike a balance between the rights of
to be interfered with.
the individual and those of the society as a whole, between
individual liberty and social control. Example: choice of what clothes to wear in different situations
Statement 2 is incorrect. Positive liberty is the 'freedom to'
77. Solution: (c)
achieve your goals and Jive life as you want.
Exp) Option c is the correct answer.
Positive liberty is the possibility of acting - or the fact
With regard to cultural and educational rights, Article of acting - in such a way as to take control of one's life
30 dearly provides that all the minorities shall have the and realize one's fundamental purposes. Positive liberty
right to establish and administer educational institutions of recognises that one can be free only in society (not outside
their choice. In fact, in Judgement 2007 Malankara Syrian it) and hence tries to make that society such that it enables
Catholic college case, Supreme Court specified that these
the development of the individual whereas negative liberty is
institutions can admit eligible students of their choice and
only concerned with the inviolable area of non-interference
to set up reasonable fee structure. They can also reserve seats
and not with the conditions in society.
for students of their community, but it nowhere mentions
that it will only allow the children of those communities The main implication of this definition is that negative
who have opened the educational institution. These cultural liberty implies a small government as there are less laws
and educational rights either talk about any section has the and constraints passed and positive liberty requires a forge
right to preserve its culture or language, or the linguistic and state which provides a welfare system and other means for
religious minorities can establish educational institutions of people to be able to achieve their goals.
their choice. This right does not mention any obligation 80. Solution: (c)
on government ·schools to ensure the children of minority
Exp) Option c is the correct answer.
groups will be introduced to their belief and culture.
Universal adult franchise means that the right to vote should
78. Solution: (b) be given to all adult citizens without the discrimination of
Exp) Option b is the correct answer. caste, class, colour, religion or sex,
Statement 1 is incorrect. In order to ensure equality of all Universal Adult Franchise is based on equality of rights
religions, Government does not levy tax on the religious which is a basic principle of democracy. It demands that

ml>----

KING R QUEEN P [ऋषभ राजपूत ]


II INDIAN POLITY AND GOVERNANCE H
the right to vote should be equally available among all. To equals equally would require that people should not be
deny any class of persons from exercising this right is to discriminated against on grounds of dass, caste, race or
violate their right to equality. In fact, the spirit of democracy gender. They should be judged on the basis of their work
can be maintained only if the people are given the right to and actions and not on the basis of the group to which they
vote without any discrimination. The exercise of right to belong. Therefore, if a male and female employee perform
vote adds to the individual's self-respect, dignity, sense of the same kind of work, they should receive the same kind of
responsibility, and political and civic education. In other reward.
words, the system of adult franchise is the bedrock of a
Pair 2 is incorrect. Proportionate justice is the second
democratic system. People are called political sovereign
principle of justice. it means rewarding people according
because they possess the right to vote a government into
to the scale and quality of effort. Although equality is an
power, or to vote a government out of power. That is why
important principle but we cannot have equal treatment for
democracy has sometimes been described as a mode of
all. How will you feel if you have worked so hard for your
appointing, controlling and dismissing governments by the
exams and you get the same marks as your friend who has
people.
slept and played during exams. You will feel this is injustice.
Right to vote is not a fundamental right in India. So, justice has to be according to the merit. So, for justice
81. Solution: (c) in society, equal treatment needs to be balanced with
proportionality. Therefore, the given example of Women
Exp) Option c is the correct answer.
Working at night requiring security arrangements for them
Statement l is correct. To achieve equality, liberals say does not best describes the proportionate justice principle.
that let state intervene in providing minimum welfare to
Pair 3 is incorrect. Recognition of special needs is the third
people. And when people have attained minimum standard
principle of justice. Everybody cannot be treated equally.
of living, allow competition between them and everybody
there are people with special needs also. for example:
will then get wealth according to their hard work. So long as
handicaps. these people have disabilities and cannot be
competition is there, inequalities will not be there in the
compared with others.
system.
These sections are not provided with the special advantages
Karl Marx says the need is to have state or public control
to a particular section but raise them to a level to compete
over the resources and property of the society.
with other sections of society on equal footing.
Statement 2 is correct. Socialist believe that social,
So, physical disabilities, lack of education and health care
economic and political inequality are Jin ke.-l F,w <"Xample,
are some factors which are considered grounds for special
rich people having control over wealth leads to control
treatment. We need to recognise that if women work at night,
over political system and this also makes them powerful in
then security issues arise and we need to take note of that
society. Unlike socialists, liberals do not believe that political,
rather than comparing them with male colleagues.
economic and social inequalities are necessarily linked. They
maintain that inequalities in each of these spheres should Giving marks according to the efforts made may be injustice
be tackled appropriately. Thus, democracy could help to to a person with special needs like a disabled person. In such
provide political equality but it might be necessary to also circumstances to make him on equal footing with others
devise different strategies to deal with social differences and extra time in writing exam or/ and reservation of posts are
economic inequalities. provided.

82. Solution: (d) 83. Solution: (d)


Exp) Option d is the correct answer. Exp) Option d is the correct answer.
Justice involves the well-being of all people. The term 'State' has been used in different provisions of the
There are the three principles of justice Constitution concerning the fundamental rights. Article 12
of the constitution has defined the term for the purposes of
a. equal treatment for equals.
Part III i.e Fundamental rights. According to it, the State
b. proportionate justice includes the following:
c. recognition of special needs (a) Government and Parliament of India, that is, executive
Pair l is correct. It is considered that all individuals and legislative organs of the Union government.
share certain characteristics as human beings. Therefore, (b) Government and legislature of states, that is, executive
they deserve equal rights and equal treatment. Some of and legislative organs of state government.
the important rights which are granted in most liberal
(c) All local authorities that is, municipalities, panchayats,
democracies today include civil rights such as the rights
district boards, improvement trusts, etc.
of life, liberty and property, political rights like the right
to vote, which enable people to participate in political (d) All other authorities, that is, statutory or non-statutory
processes, and certain social rights which would include the authorities like LIC, ONGC, SAIL, etc.
right to enjoy equal opportunities with other members of Thus, State has been defined in a wider sense so as to
the society. Apart from equal rights, the principle of treating include all its agencies. It is the actions of these agencies that

Practicebook----------------------------------------------alm

KING R QUEEN P [ऋषभ राजपूत ]


II INDIAN l'OU'.JJY.ANDGOVERNANCE II
can be challenged in the courts as violating the Fundamental 86. Solution: (d)
Rights. According to the Supreme Court, even a private Exp) Option d is the correct answer.
body or an agency working as an instrument of the State
Statement a is incorrect. All Fundamental rights guarantee
falls within the meaning of the 'State' under Article 12.
the rights of an individual against the state but not all are
84. Solution: (d) available against the action of private individuals. For
example, the first provision of Article -15 that the state
Exp) Option dis the correct answer.
shall not discriminate against any citizen on the grounds
Statement l and 2 are incorrect. The difference between the of religion, race, caste, sex or place of birth prohibits
procedure established by law and due process of law is that discrimination only by state and not by private individuals.
the due process of law gives wide scope to the Supreme Court While the second provision of not allowing citizens access to
to grant protection to the rights of its citizens. It can declare shops, public place on the basis of religion, race, caste, sex
laws violative of these rights void not only on substantive or place of birth prohibits discrimination both by state and
grounds of being unlawful, but also on procedural grounds by private individuals. Article 19 are protected only against
of being unreasonable. state and not against private individuals.
Procedure Established by Law means that a law that is duly Statement b is incorrect. These rights are binding upon all
enacted by the legislature or the concerned body is valid if it the authority that has got power to make laws whether it be
has followed the correct procedure. Following this doctrine central government or state government or local government.
means that, a person can be deprived of his life or personal Statement c is incorrect. The role of guiding the Indian
liberty according to the procedure established by law. Our state to promote social and economic welfare is the function
Supreme Court, while determining the constitutionality of DPSPs (Directive principles of state policy) and not of
of a law, however examines only the substantive question Fundamental rights.
i.e., whether the law is within the powers of the authority Statement dis correct. Both the ,upreme court under Article
concerned or not. It is not expected to go into the question 32 and High courts under Article 226 can issue writs for the
of its reasonableness, suitability or policy implications". enforcement of Fundamental rights. However, a person can
Statement 3 is incorrect. The scope of judicial review power go directly to Supreme court for his violation of fundamental
of the Supreme Court in India is narrower than that of what rights. It is not necessary that he can go to Supreme court
exists in US. This is because the American Constitution only by the way of appeal mechanism.
provides for 'due process of law' against that of 'procedure 87. Solution: (a)
established by law' contained in the Indian Constitution.
Exp) Option a is the correct answer.
85. Solution: (b) Option a is correct. Article 14 provides that the state sirnll
Exp) Option bis the correct answer. not deny to any person equality before the law (ll' equal
protection of laws within the territory of lndia. So, denying
The Fundamental Rights are enshrined in Part III of the
the promotion of women officers is the rule ,)f faw
Indian Constitution from Articles 12 to 35. The Fundamental
this article sees all person equal in the eye of law. Article 15
Rights are guaranteed by the Constitution to all persons states that the state shall not discriminate against any citizen
without any discrimination. They uphold the equality of all on grounds only of religion, race, rnste, sex, place of birth
individuals, the dignity of the individual, the larger public or any of them. But denying the promotion on the bash of
interest and unity of the nation. sex (women) is a violation of article l S. Axtide l.6 slates that
Statement 1 is incon-ect. Article 33 of the Constitution no citizen shall, on grounds only of religion, nice, nrnte, sex,
provides that Parliament (and not a state legislature) is descent place of birth, residence or any of them be ineligible
empowered to restrict or abrogate the Fundamental Rights for or discriminated against in respect of any employment
of the members of armed forces, members of forces charged or office under the state. Thus, promotion if only provided
with maintenance of public order, intelligence & counter- to men and not to women is a clear violation of Article 16.
intelligence agencies, paramilitary forces, state police forces. Right to undertake any profession- All citizens are given
For example, Police: non-gazetted personnel from the post of the right to practise any profession or to carry on any
constable to Inspector are not allowed to form any kind of occupation, trade or business. This right only covers all the
union or association under the Police Forces (Restriction of means of earning one's livelihood. It does not prohibit any
Rights) Act, 1966, discrimination in respect of any employment or office under
the state (this is dealt in Article 16).
The power to make laws under Article 33 is conferred only
on Parliament and not on state legislatures. 88. Solution: (b)
Statement 2 is correct. Any law made to restrict the Exp) Option b is the con-ect answer
fundamental rights of members of armed forces cannot be Statement l is incorrect. The right conferred on minorities
challenged in any court on the ground of contravention of under Article 30 is only to ensure equality with the
any of the Fundamental Rights. majority and not intended to place the minorities in a more

KING R QUEEN P [ऋषभ राजपूत ]


II INDIAN POLITY AND GOVERNANCE II
advantageous position vis-a--vis the majority. There is no Statement l is correct. Article 16 of the constitution
reverse discrimination in favonr of minorities. provides for equality of opportunity for all citizens in
Statement 2 is correct. As per the second provision of matters of employment or appointment to any office under
Article 29 of India constitution, no citizen shall be denied the State. Article 16(4) of the constitution explicitly clarifies
admission into any educational institution maintained by that a policy like reservation will not be seen as a violation of
the Stale or receiving aid out of State funds on grounds right to equality. The state is permitted to make reservation
only of religion, race, caste, or language. This feature is not in favour of, any backward class, if they are not adequately
necessarily restricted to minorities only, as it is c:ommonly represented.
assumed to be but extended to all citizens in the country. Statement 2 is incorrect. The text of the Preamble of the
ConF.titution of Tndia explicitly provides for equality of
89. Solution: (a) status and equality of opportunity to all its citizens.
Exp) Option a is the correct answer.
Important Tips
Affirmative action refers to a set of policies and practices
The Preamble -
within a government or organization seeking to inuease
the representation of backward groups based on their WE, THE PEOPLE OF INDIA, having solemnly resolved
gender, race, sex1rnlity, cree<l or nationality in areas in to constitute India into a SOVEREIGN SOCIALIST
SECULAR DEMOCRATIC REPUBLIC and to secure to
which they are underrepresented such as education and
all its citizens:
employment.
JUSTICE, social, economic and political; LIBERTY of
Affirmative actions in India can be categorized into 3 main
thought, expression, belief, faith and worship;
dimensions, which are given below.
EQUALITY of status and of opportunity;
Appointment and promotion in government services.
(Statement l is correct) and to promote among them all FRATERNITY assuring
the dignity of the individual and the unity and integrity
Reservations in public education institutions. (Statement of the Nation;
2 is correct)
IN OUR CONSTITUENT ASSEMBLY this twenty-sixth
Reservation of seats in Central and State legislatures. day of November, 1949, do HEREBY ADOPT, ENACT
The purpose of affirmative action is to establish fair access AND GIVE TO OURSELVES THIS CONS'fJ'T'TJTION.
to employment opportunities to create a workforce that is
92. Solution: (d)
an accurate reflection of the demographics of the qualified
available workforce in the relevant job market. Exp) Option d is the correct answer.
Every child to get free education cannot be called as Statement 1 is incorrect. The 44th AnH,ndment Act 1978 of
affirmative action, as it is provided to every child irrespective the Constitution of India inserted a 1ww artkle 300A titled
of his/her gender, race, sexuality, caste or nationality. It is an as Right to Property which wa, inade a Constitutional right
equality of opportunity. (Statement 3 is incorrect) under this article. Though Ri1;ht to property is n'moved as
a Fundamental Right but it is still present as a Constitutional
90. Solution: (c) right. Article 300A requires the 5tatc to follow due procedure
Exp) Option c is the cot·.t'ecl answer and authority of law to deprive a person of his or hier privale
property. This article provides restrictions on the Stat:1; that
Statement l is correct: - Article ?.9 in the 1,on.~tit11tion of
it cannot take anybody'~ property wfthout the force of law
India provides for the Protection of interests of minorities.
also interpreted can be deprived of the force of law. The word
Article 29 provides that any section of the citizens residing of law here means a validly enacted law which is just, fair,
in any part of India having a distinct language, script or and reasonable.
culture of its own, shall have the right to conserve the same.
Statement 2 is incorrect. In 1978, 44 11' amendment Act to
Statement 2 is correct: - No citizen shall be denied admission the constitution removed the right to property from the list
into any educational institution maintained by the State or of fundamental rights and converted to a simple legal right
receiving aid out of State fonds on grounds only of religion, under article 300A. Necessary amendments for this purpose
race, caste, or language (Article 29). were made to article 19 and article 31. It would, however,
Statement 3 is correct: - Cultural and Educational Rights be ensured that the removal of property from the list of
safeguards the rights of linguistic, cultural, and religious fundamental rights would not affect the right of minorities
minorities .' rticles 29 and 30 deals with the cultural and to establish and administer educational institutions of their
educatio;.. rights of Indian citizens. Fundamental Rights choice.
under Article 15, 16, 19, Article 29 and Article 30 are
93. Solution: (b)
available only to Citizens of India and not to foreigners.
Exp) Option b is the correct answer,
91. Solution: (a) Article 12 defines the term 'State' for the purpose of different
Exp) Option a is the correct answer. prnvisiom eonct:rning the fundamental rights.

Pradicebook-------------------------------------------
B,lFo.t'Ulll~

KING R QUEEN P [ऋषभ राजपूत ]


II INDIAN POLITY AND GOVERNANCE II
Statement l is correct: The Supreme Court has held that the 95. Solution: (a)
term 'state' is wide enough to include all authorities created Exp) Option a is the correct answer.
by the Constitution or statute on whom powers are conferred
The rule of equality before law is not absolute and there are
by law. Thus, even a private body or an agency working as
constitutional and other exceptions to it.
an instrument of the State falls within the meaning of the
'State' under Article 12. It is not necessary that the statutory Statement l is incorrect: No criminal proceedings shall
authority should be engaged in performing governmental or be instituted or continued against the President or the
sovereign function. Governor in any court during his term office. However civil
proceedings can be instituted after the expiration of two
Statement 2 is correct: All local authorities, that is,
months of delivery of notice. Thus, no civil proceedings
municipalities, panchayats, district boards, improvement
against the President or the Governor shall be instituted
trusts, etc. are included under the definition of state under
during his term of office in any court in respect of act clone
Article 12. According to List II of VII Schedule 'local
government' includes a municipal corporation, urban by him in his personal capacity, whether before or after he
improvement trust, <lhtl'l,t ho:ir<l.~, ml11i113 settlement entered upon his office, until the expiration of two months
authorities and other local authorities for the purpose of next after notice has been delivered to him.
local self-government or village administration. Statement 2 is incorrect: Article 31-C is an exception to
Statement 3 is incorrect: In Article 12, the expression 'other Article 14. It provides that the laws made by the state for
authorities' is used after mentioning the Government, implementing the Directive Principles contained only in
Parliament of India, the Government and Legislature of each clause (b) or clause (c) of Article 39 (and not complete
of the State and all local authorities. The Madras High Court Article 39) cannot be challenged on the ground that they are
has held that 'other authorities' could only mean authorities violative of Article 14, not Article 21.
exercising governmental or sovereign functions. It cannot Statement 3 is correct: The foreign sovereigns (rulers),
include persons, natural or juristic, such as, a university ambassadors and diplomats enjoy immunity from both
unless it is 'maintained by the State'. criminal and civil proceedings. The UNO and its agencies
enjoy the diplomatic immunity
94. Soluti.on: (b)
Exp) Opt.ion h i6 the correct answer. 96. Solution: (b)
Ruk of iaw rneam; that no man is above law and also that Exp) Option b is the correct answer.
every person i:i subject to the jurisdiction of ordinary courts Right to assemble peaceably and without arms is one of the
of law irrespective of t!wir position and rank. six rights guaranteed by Article 19 of the Constitution to
StakILen!: l is cmTe.:t: In .Keshavananda Bharti v. State of all citizens. It includes the right to hold public meetings,
Kerala, the Supreme Court !ms included the Rule of Law as demonstrations and take out processions.
the bar;k feature of !he C,,m:tltution, In this case, though
Statement l is incorrect: An assembly of five or more
the Supreme Court upheld tlw ;1mendlng power of the
persons becomes unlawful if the object is to resist the
Parliament 'Nhkh extends to every Article provided under
execution of any law or legal process under Section 141
the Constitution but has limited thal po•Ncr by providing that
of the Indian Penal Code (and not CrPC). Under Section
such power cannot he used in aniending the basic feature of
144 of Criminal Procedure Code (1973), a magistrate can
the Constitution.
restrain an assembly, meeting or procession if there is a risk
Statement 2 is correct: T\1ere ar,~ certain exceptions to rule of obstruction, annoyance or danger to human life, health or
of law provided under the Imfom Constitution and other safety or a disturbance of the public tranquillity or a riot or
laws. Public officials like ministers, local authorities, public any affray.
officers have many powers, immunities and privileges which
ordinary citizens don't have. For example, the President/ Statement 2 is correct: Under Section 141 of the Indian
Governor is not answerable to the court of law in discharge Penal Code, as assembly of five or more persons becomes
of his executive functions. unlawful if the object is to resist the execution of any law or
legal process. Other grounds for imposing restrictions are:
No criminal proceedings whatsoever can be instituted
if the object is
against President or Governor of state, while he is in office.
to forcibly occupy the property of some person
Statement 3 is incorrect: In India, Constitutional Supremacy
prevails where the Doctrine of Separation of Powers has been to commit any mischief or criminal trespass
held which permits the review of the legislative acts. to force some person to do an illegal act
Rule oflaw has no fixed articulation in the Indian constitution to threaten the government or its officials on exercising
though the Indian courts refer to this phrase in variety of its lawful powers
judgements.
97. Solution: (b)
Indian constitution is the law of the land and prevails over
Judiciary, the Legislature and the Executive. These three Exp) Option b is the correct answer.
urgam uf lhe slate have to act according to the principles Article 15 provides that the State shall not discriminate
engraved in the constitution. against any citizen on grounds only of religion, race, caste,

Ille---------------------- --------Practice book

KING R QUEEN P [ऋषभ राजपूत ]


II INDIAN POUTY AND GOVERNANCE II
sex or place of birth. The use of the word 'only' connotes The right to protest involves the exercise of two fundamental
that discrimination on other grounds is not prohibited. rights: freedom of speech and expression under Article 19
Option l is correct: The state is permitted under Article 15 (1) (a) of the Constitution and the freedom to assemble
to make any special provision for the advancement of any peacefully under 19 ( 1) (b).
socially and educationally backward classes of citizens or for Article 19(l)(a) of the Indian constitution "Freedom of
the scheduled castes and scheduled tribes. For example, speech and expression" is a fundamental right given to its
reservation of seats or fee concessions in public educational citizen against the state, which includes carrying out the
institutions. peaceful public demonstration.
Option 2 is correct: The state is empowered to make any Article 19( 1) (b) provides for the right to assemble peaceably
special provision for the advancement of any socially and without arms. This includes the right to hold public
and educationally backward classes of citizens or for the meetings, hunger strikes, and the right to take out
scheduled castes or the scheduled tribes regarding their processions.
admission to educational institutions including private
100. Solution: (d)
educational institutions, whether aided or unaided by the
state, except the minority educational institutions. In Exp) Option d is the correct answer.
order to give effect to this provision, the Centre enacted the Article 30 of the Indian Constitution states the right
Central Educational Institutions (Reservation in Admission) of minorities to establish and administer educational
Act, 2006, providing a quota of 2 7% for candidates belonging institutions. It says: "All minorities, whether based on
to the Other Backward Classes (OBCs) in all central higher religion or language, shall have the right to establish and
educational institutions. administer educational institutions of their choice:'
Option 3 is correct: The state is permitted to make any Statement 1 is incorrect. Any minority educational
special provision for women and children. For example, institutions (MEis) which seek recognition from the state
reservation of seats for women in local bodies or provision but not aid from the state are subjected to regulatory
of free education for children. power of state with regard to syllabus prescription, academic
Option 4 is incorrect: The state is empowered to make any standards, discipline, sanitation etc. MEis which neither
special provision for the advancement of any economically seek recognition from the state nor aid from the state
weaker sections of citizens. The Parliament of India can manage their own affairs, but general laws, labour law,
enacted the Constitution (103rd Amendment Act,2019) taxation law, industrial law etc still apply to these institutions.
and authorized the State to make reservations in higher Statement 2 is incorrect. The RTE 2009 mandates all
education and affairs of Public Employment solely on the private schools to reserve 25 per cent seats to students from
basis of economy. The act amended Article 15 and 16 of the economically weaker sections at the entry level.
Constitution by inserting 15(6) and 16(6). The amendment The Supreme \.011rt in Snc:i,:ty for Un-Aided Private
was made to provide for 10% reservaliu11 Lu Lhe citizens, Schools of Rajasthan v. Union of India held that the Right
belonging to the Economic Weaker Section (E\A/S) category. of Children lo Free and Compulsory Jlducatlon Act, 2009
is not applicable to unaided minority schools. Thus, MEis
98. Solution: (a)
are not required to provide admission to children in the age
Exp) Option a is the correct answer. group of 6-14 years up to 25% of enrolment reserved for
Article 16 provides for equality of opportunity for all economically backward section of society
citizens in matters of employment or appointment to any Statement 3 is incorrect. The Supreme Court has ruled
office under the State. No citizen can be discriminated on that the National Eligibility cum Entrance Test (NEET) for
grounds of only religion, race, caste, sex, descent, place of admissions to these courses would also apply to minority -
birth or residence. both aided and unaided and private educational institutions.
Statement l is correct: Article 16(2) states that "no citizen The Constitution provides a limitation on the power of the
shall, on grounds only of religion, race, caste, sex, descent, State to interfere with life, liberty and rights however, the
place of birth, residence shall be discriminated against any concept of limited government cannot be extended to a level
employment or office under the State". The provision is when it defeats national interest. So, providing central
supplemented by the other clauses in the Constitution that examination in the form of NEET cannot be said to be
guarantee equality. violation of the rights under Articles 19(l)(g) and 30 of the
Statement 2 is incorrect: However, Article 16(3) of the Constitution.
Constitution provides an exception by saying that Parliament,
101. Solution: (b)
not State legislatures may make a law "prescribing" a
Exp) Option b is the correct answer.
requirement of residence for jobs in a particular state. This
power vests solely in the Parliament, not state legislatures. The Padma Awards are one of the highest civilian honours
of India announced annually on the eve of Republic Day.
99. Solution: (b) The Awards are given in three categories: Padma Vibhushan
Exp) Option bis lhe i.:orred answer. (for exceptional and distinguished service), Padma Bhushan

Practicebook------------------------------------------------<111

KING R QUEEN P [ऋषभ राजपूत ]


II INDlANPOLITY ANDGOVERNANCE II
(distinguished service of higher order) and Padma Shri -the Convention on Diplomatic Relations, 1961, and the
(distinguished service). The award seeks to recognize Convention on Consular Relations, 1963.
achievements in all fields of activities or disciplines where Statement l is correct. Diplomatic immunity is one of the
an element of public service is involved. principles of international law which limits the degree to
Statement l is correct: The Government of India instituted which the officials and employees of foreign governments
two civilian awards-Bharat Ratna & Padma Vibhushan in would be subject to the other country's authority of police
1954. The latter had three classes, namely Pahela Varg, Dusra officers and judges.
Varg and Tisra Varg. These were subsequently renamed Vienna Convention of 1961 under Article 34 lays down
Padma Vibhushan, Padma Bhushan and Padma Shri in 1955. those diplomatic agents shall be exempted from all duties
Statement 2 is incorrect: Bharat Ratna is the highest and Taxes imposed, whether personal or real, national,
civilian award of the country. It is awarded in recognition of regional, or Municipal. The sending State and the head of
exceptional service/performance of the highest order in any the mission shall be exempt from all national, regional
field of human endeavour. It is treated on a different footing or municipal dues and taxes in respect of the premises
from Padma Award. The recommendations for Bharat Ratna of the mission, whether owned or leased, other than
are made by the Prime Minister to the President of India. No such as represent payment for specific services rendered.
formal recommendations for Bharat Ratna are necessary. Statement 2 is incorrect. The Vienna Convention classifies
Statement 3 is correct: Padma Awards, which were diplomats according to their posting in the embassy,
instituted in the year 1954, is announced every year on the consular or international organisations such as the UN.
occasion of Republic Day except for brief interruption(s) A nation has only one embassy per foreign country, usually
during the years 1978 and 1979 and 1993 to 1997. in the capital, but may have multiple consulate offices,
These awards were discontinued by Janta Party government generally in locations where many of its citizens live or visit.
which came to power in 1977, headed by Morarji Desai, but Diplomats posted in an embassy get immunity, along with
were again revived by Indira Gandhi government. his or her family members.
Statement 4 is incorrect: All nominations received for The members of the family of a diplomatic agent forming
Padma Awards are placed before the Padma Awards part of his household shall, if they are not nationals of the
Committee, which is constituted by the Prime Minister receiving State, enjoy the privileges and immunities specified
every year. The Padma Awards Committee is headed by the in Articles 29 to 36 of the Vienna Convention.
Cabinet Secretary and includes Home Secretary, Secretary 104. Solution: (d)
to the President and four to six eminent persons as members.
Exp) Option dis the correct answer.
The recommendations of the committee are submitted to the
Prime Minister and the President of India for approval. 'Habeas Corpus' literally means "to have the body of". This
writ is used to release a person who has been unlawfully
102. Solution: (a) detainee! or imprisoned. By virtue of this writ, the Court
Exp) Option a is the correct answer. directs the person so detained to be brought before it to
Article 18 of Constitution of India abolishes titles and examine the legality of his detention.
prohibits the government from bestowing any title on any Habeas Corpus cannot be issued in the following cases: 1)
citizen or foreigner (except military or academic distinction) When detention is lawful 2) When the proceeding is for
contempt of a legislature or a court 3) Detention is by a
Statement 1 is correct: The hereditary titles of nobility
conferred by colonial States, such as Maharaj a, Raj Bahadur, competent court 4) Detention is outside the jurisdiction of
Rai Bahadur, Rai Saheb, Dewan Bahaclur, and so on, are the court.
prohibited by Article 18 because they violate the principle The Supreme Court held that the declaration of martial law
of equal status for all. does not ipso facto result in the suspension of the writ of
habeas corpus
Statement 2 is incorrect. In 1996, the Supreme Court upheld
the constitutional validity of the National awards. The A writ of habeas corpus will not lie and will be refused by a
Supreme Court said that National Awards do not amount to court:
'titles' within the meaning of Article 18 that prohibits only where the person or authority against whom habeas
hereditary titles of nobility. corpus is sought is not within the territorial jurisdiction
Therefore, they are not violative of Article 18 as the theory of the court.
of equality does not mandate that merit should not be Where imprisonment or detention of a person is in
recognized. However, they should not be nsed as suffixes or accordance with a decision rendered by a court of law or
prefixes to the name of the awardees by an authority in accordance with law.

103. Solution: (b) 105. Solution: (b)


Exp) Option b is the correct answer. Exp) Option bis the correct answer.
Diplomatic immunity is granted on the basis of two Article 22 provides for the Protection against arrest and
conventions popularly called the Vienna Conventions detention in certain cases.

KING R QUEEN P [ऋषभ राजपूत ]


II INDIAN FOUTY AND GOVERNANCE II
Statements 1, 2 and 4 are correct To enforce a contractual obligation;
Article 22 confers the following rights on a person who is Against the president of India or the state governors; and
arrested or the detained under the ordinary law: (i) The Against the chief justice of a high court acting in judicial
right to be informed of the grounds of arrest. (ii) The right capacity.
to consult and to be defended by the legal practitioner(iii)
The right to be produced before the magistrate within a 108. Solution (c)
period of twenty-four hours (iv) The right to be released afler Exp) Option c is the correct answer.
24 hours unless the magistrate authorises further detention. Statement l is correct. Article 22 (1) of the Indian
Statements 3, 5 and 6 are incorrect Constitution and Section-SO, of CrPc 1973 states that no
Section- 50 (2) of CrPC provides that the arrested person person who is arrested shall be detained in custody without
has the right to get released on bail by making arrangement being informed of the offense committed by the person
for the sureties. being arrested.
Article- 39A(DPSP) provides free legal aid to people in Statement 2 is correct. ln The Case of Nandini Sathpathy
need. V. P.L.Dani, SC stated an accused has the right to remain
silent during the process of interrogation. It highlights
The right to keep quiet does not have any mention in any
Article-20(3) of the Indian Constitution, which says
Indian law, however, its authority can be derived from Cr PC
that no person accused of any offense shall be compelled
as well as the Indian Evidence Act.
to be a witness against himself i.e., the principle of Self-
106. Solution: (d) Incrimination. That means the accused has a right to remain
Exp) Option d is the correct answer. silent.

Article 23 provides for the Prohibition of traffic in human Statement 3 is correct. Article 22(2) of the Indian
hPings and forced labour. Constitution states that every person who is arrested and
detained in custody shall be produced before the nearest
Article 23(1): Traffic in human beings and the beggar and
magistrate within a period of twenty-four hours of arrest
other similar forms of forced labour are prohibited and any
excluding the time necessary for the journey from the place
contravention of this provision shall be an offence punishable
of arrest lo lhe LULU l uf Llie rnagislrale.
in accordance with the law.
Statement 4 is incorrect. According to Section 26 of the
Statement 1 is incorrect: Article 23 does not prevent the Indian Evidence Act, any confession made by any person
state to impose compulsory services for public purposes in the police custody cannot be used as evidence either
like military service or social service. For this, the state is against the accused or in favour of the accused for any legal
not bound to pay. release until and unless it has been proved in the immediate
Statement 2 is incorrect: Article 23 protects citizens not presence of a Magistrate as against or in favour of a person
only against the State but also from private citizens. The accused.
State is obliged to protect citizens from these evils by taking
punitive action against perpetrators of these acts (which are 109. Solution (a)
considered crimes), and also lalu, vu~ilive aclium tu abolish Exp) Opliun a fo lhc correct answer.
these evils from society Option a is correct: Article 14 says that the State shall not
Statement 3 is incorrect. In making any such service deny to any person equality before the law or the equal
compulsory for public purposes, the State, however, cannot protection of the laws within the territory of India. The
make discrimination on the basis of religion, race, caste or discrimination in providing citizenship based on religion,
class or any of them. by government, is an infringement of Article 14 where the
government discriminated on the basis of religion thereby
107. Solution: (c) violating equality amongst Indians.
Exp) Option c is the correct answer.
110. Solution (a)
Statement l is correct: Mandamus literally means 'we
Exp) Option a is the correct answer.
command'. It is a command issued by the court lo a public
official asking him to perform his official duties that he Under the Indian constitution, Fundamental Rights are
has failed or refused to perform. It can also be issued against mentioned in the Part 3 of Articles from 12 to 35 while
any public body, a corporation, an inferior court, a tribunal directive principles are mentioned in part 4 of article 36
or government for the same purpose. to 5
Statement 2 is correct: The writ of mandamus cannot be Statement 1 is incorrect: Directive Principles of State
issued Policy aim to create social and economic democracy.
Whereas Fundamental rights aim at establishing political
• Against a private individual or body;
democracy in the country by preventing the establishment
To enforce departmental instruction that does not of an authoritarian and despotic rule in the country and
poasess statutory force; protecting the liberties and freedoms of the people against
When the duty is discretionary un<l nut mandatory; the invasion by the Stale.

------11111
KING R QUEEN P [ऋषभ राजपूत ]
II INDIAN POLITY AND GOVERNANCE

Statement 2 is correct: Fundamental Rights are justiciable Option a is incorrect: This statement defines the term 'state'
in nature. But the directive principles are non-justiciable in the context of the discipline of Political Science. But this
in nature, that is, they are not enforceable by the courts for is not the definition of term 'state' that is defined by Indian
their violation. constitution.
Statement 3 is incorrect: Fundamental rights do not Option c is incon-ect: This statement defines the term
require any legislation for their implementation. Many nation. A nation is a community of people formed on the
of the FRs are automatically enforced. Whereas, Directive basis of a common language, territory, ethnicity etc.
principles require legislation for their implementation. They
Option d is incorrect: A defined territory that forms a
are not automatically enforced.
community with shared culture and ruled by sovereign body
111. Solution (a) defines the term 'country'.
Exp) Option a is the correct answer.
113. Solution (d)
Option 1, 2 and 3 are correct: Article 12 has defined the
Exp) Option d is the correct answer.
term for the purposes of Part III. According to it, the State
includes the following: Statement l is incorrect: Article 13 declares that all laws
that are inconsistent with or in derogation of any of the
Government and Parliament of India, that is, executive
and legislative organs of the Union government. fundamental rights shall be void. Parliament and the state
legislatures are clearly prohibited from making laws
Government and legislature of states, that is, executive
that may take away or abridge the fundamental rights
and legislative organs of state government. (Hence
Statement 3 is correct) guaranteed to the citizen. The Parliament can curtail or
repeal them but only by a constitutional amendment act
All local authorities, that is, municipalities, district
and not by an ordinary act. Moreover, this can be done
boards, improvement trusts, gram panchayats etc.
without affecting the 'basic structure' of the Constitution.
(Hence Statement l is correct)
All other authorities, that is, statutory or non-statutory Statement 2 is inconect: Article 35 lays down that the power
authorities like LIC, ONGC, SAIL, etc. (Hence to make laws to give effect to certain specified fundamental
Statement 2 is correct) rights shall vest only in Parliament and not in the state
legislatures.
Option 4 and 5 are incorrect: Political Parties as such do not
come under the definition of 'state' under article 12 When Statement 3 is incorrect: Article 13 declares that all laws
the Supreme Court takes judicial decisions, it does not come that are inconsistent with or in derogation of any of the
within the meaning of State. fundamental rights shall be void. Temporary laws like
ordinances issued by the President or the State Governors
Important Tips
can be challenged in the courts as violation of Fundamental
In Rupa Ashok Hurra v Ashok Hurra case, the Supreme
Right and hence, can be declared as void.
Court reaffirmed and ruled that no judicial proceeding
could be said to violate any fundamental right. It was said 114. Solution (a)
that when they give judicial decisions, Supreme Court do
Exp) Option a is the correct answer.
not con1e within the meaning of State. But, while courts
perform their administrative function, they are within the Statement l is correct: The expression 'equality before
State's definition. law' stems from the English Common Law (British) and
the expression 'equal protection of law' stems from the
112. Solution (b) American Constitution. The concept of 'equality before law'
Exp) Option b is the correct answer. is an element of the concept of 'Rule of Law', propounded
Under the Indian constitution, Article 12 defines the by A.V. Dicey, the British jurist. While the concept of 'equal
term 'State' as used in different Articles of Part HI of the protection of laws' has been taken from the American
Constitution. Constitution.
It says that unless the context otherwise requires the term Statement 2 is incorrect: The expression 'equality before
'State' includes the following; - I.aw' is a negative concept as it implies an absence of
The Government and Parliament of India, i.e., Executive special privileges that favour any individual. However,
and Legislature of the Union. the expression 'equal protection of law' on the other
The Government and Legislature of each State, i.e., hand, is a positive concept as it simply means that there
Executive and Legislature of State. should be equality of treatment of individuals in similar
All local and other authorities within the territory of circumstances.
India. Statement 3 is incorrect: Indian Constitution has accepted
All local and other authorities under the control of the both the concept of 'equality before law' and 'equal
Government of India. protection of laws' under Article 1

1B1---------------------------------------------Practicebook

KING R QUEEN P [ऋषभ राजपूत ]


H INDIAN POLITY AND GOVERNANCE II
IJ.5. Solution (c) to give thumb impression, specimen signature, blood
Option c is the crnrrect answer. specimens, and (iii) compulsory exhibition of the body.
Further, it extends only to criminal proceedings and not to
are reasonable daims of persons recognized by
civil proceedings or proceedings which are not of criminal
and sanctioned law.
nature.
Rights are daims of a person over other fellow beings,
over the society and over the government. 117. Solution (a)

Rights acquire meaning only in society. What is Exp) Option a is the correct answer.
recognized by Gocicty u;; rightful becomes ll1e L>asis of Article 1.1. grants protection to persons who are aaesled or
rights. That is why the notion of rights chane;Ps from time detained. Detention is of two types, JJarnely, punitive and
to tirne and society to society. preveulive.
When the socially recognized claims are written into Statement l is correct: Preventive detention means
law, they acquire real force. When law recognizes some detention of a person without trial and conviction by a
claims, they become enforceable. We can then demand court. Its purpose is not to punish a person for a past offence
their application. When fellow citizens or the government but to prevent him from committing an offence in the near
do not respect these rights, we call it violation or future.
infringement of our rights. In such circumstances citizens Statement 2 is incorrect: The 44th Amendment act of 1978
can approach courts to protect their rights. has reduced the period of detention without obtaining the
opinion of the advisory board from 3 months to 2 months.
Important Tips
However, this provision has not yet been brought into force,
Kinds of Rights hence, the original period of three months still continues.
Civil Rights: These arc basic rights that protect Also, if an advisory board reports sufficient cause for an
individuals' freedom and equality, including the right extension, its 3--month period can also be extended.
to life, liberty, and property. They ensure fair treatment
Statement 3 is incorrect: The Constitution has divided the
and protection from discrimination.
legislative power with regard to preventive detention between
Natural Rights: Natural rights, also known as inherent the Parliament and the state legislatures. The Parliament has
rights or unalienable rights, are fundamental rights exclusive authority to make a law of preventive detention
that are believed to be inherent to all individuals by for reasons connected with defence, foreign affairs and the
virtue of their humanity. security of India. Both the Parliament as well as the state
Social Rights: Social rights guarantee access to legislatures can concurrently make a law of preventive
essential services and benefits, such as education, detention for reasons connected with the security of a state,
healthcare, housing, and social security, to ensure a the maintenance of public order and the
minimum standard of living for all members of society.
, --·~-- -·--·--·----·~-- US. Solution (b)
H6. Solution (d) Exp) Option bis the correct answer.
Exp) Option d is the correct answer. Most of Fundamental Rights in Indian Constitution are
Article 20 of Indian Constitution grants protection against available against the arbitrary action of the State, but some
arbitrary and excessive punishment to an accused person. are also available against the action of private inniviriuals.
SLalement l ls i11correct: Article '.W(prohibits enactment of Option 1 and 4 are correct:
ex-post-facto laws (that imposes penalties retrnspectively). Right to Equality Before Law (Article 14) of the Indian
However, this limitation is imposed only on criminal laws Constitution guarantees that no person shall be denied
and not on civil laws or tax laws. In other words, civil the right to equality before law or the equal protection
liability or a tax can be imposed retrospectively. of law in the territory of India. This is a right that can
Statement 2 is incorrect: Article 20 states that- No person be claimed by any person, whether a citizen or a non-
shall be prosecuted and punished for the same offence citizen, on Indian soil. It is available only against the
more than once. The protection against double jeopardy is state and not against the private individual.
available only in proceedings before a court of law or a Article 22 (Right to Protection Against Arrest and
judicial tribunal. It is not available in proceedings before Detention) deals with the protection against arrest and
departmental or administrative authorities as they are not detention in certain cases. This article is applicable to
of a judicial nature. both citizens and non-citizens. This provision extends
certain procedural safeguards for individuals in case of
Statement 3 is incorrect: Article 20(states that no person
an arrest. It is available only against the state and not
accused of any offence shall be compelled to be a witness
against the individual. •
against himself (no self-incrimination). The protection
against self-incrimination extends to both oral evidence and Option 2 and 3 are incorrect:
documentary evidence. However, it does not extend to (i) Prnhibition of traffic in human beings and forced
compulsory pwJucLion of material objects, (ii) compulsion labour (Article 23) explicitly prohibits human

Practice b o o k - - - - - - - -
------------DI
KING R QUEEN P [ऋषभ राजपूत ]
II INDIAN POLITY AND GOVERNANCE II
trafficking, child labor, untouchability, and various other 121. Solution (c)
activities related to it. It protects the individual not only Exp) Option c is the correct answer.
against the State but also against private persons.
Article 31 C states that no law which seeks to implement the
Abolition of Untouchability (Article 17) abolishes socialistic Directive Principles specified in Article 39(b) and
'untouchability' and forbids its practice in any form. The 39 (c) (shall be void on the ground of contravention of the
Supreme Court held that the right under Article 17 is Fundamental Rights conferred by Article 14 (equality before
available against private individuals. law and equal protection of laws) and Article 19 (protection
of six rights in respect of speech, assembly, movement, etc.).
119. Solution (b)
Thus, it gives primacy to the implementation of the Directive
Exp) Option b is the correct answer. Principles specified in Article 39 (and (over Fundamental
Statement l is correct: Fundamental Rights promote the Rights under article 14 and 19.
welfare of the individual. They promote the ideal of political
122. Solution (c)
democracy. The Directive Principles of States Policy
promotes the welfare of the community. They promote the Exp) Option c is the correct answer.
ideal of social and economic democracy. The Harm Principle says that people should be free to act
Statement 2 is correct: Fundamental Rights are negative as as they wish unless their actions cause harm to somebody
they restrict the power of the state. The Directive Principles else. The Harm Principle means that there should be a
of States Policy are positive as they require the State to take constraint on a person from acting in a way that may cause
certain steps. harm to others. It was first proposed by English philosopher
John Stuart Mill. It means people should be free to act in
Statement 3 is incorrect: Judiciary can declare a law as the way they want, however their actions should not cause
unconstitutional and invalid if it violates the Fundamental harm to somebody else. Phrase "Your freedom to swing your
Rights. The courts cannot declare a law violative of any fist ends where my nose begins" reflects the sentiment of the
of the Directive Principles as unconstitutional and invalid. harm principle.
However, they can uphold the validity of a law on the ground
that it was enacted to give effect to a directive. Important Tips
According to 'Loss Aversion' Theory, if two choices are
120. Solution (a) put before an individual, both equal, with one presented in
Exp) Option a is the correct answer. terms of potential gains and the other in terms of possible
Option a is correct: Article 25 says that all persons are losses, the former option will be chosen.
equally entitled to freedom of conscience and the right to
123. Solution (b)
freely profess, practice and propagate religion. The right
to propagate does not include a right to convert another Exp) Option b is the correct answer.
person to one's own religion. Forcible conversions impinge Rule of law is a concept propounded by A. V. Dicey.
on the 'freedom of conscience' guaranteed to all the Statement 1 is incorrect. The concept of Rule of Law is
persons alike. borrowed from the British Constitution.
Option b and c are incorrect: The Supreme Court held that Statement 2 is correct. In Indira Nehru Gandhi V Raj
the freedom of speech and expression includes the Right to Narayan case, the Supreme Court held that the 'Rule of
propagate one's views as well as the views of others and Law' as embodied in Article 14 is a 'basic feature' of the
Right to demonstration or picketing but not right to strike. constitution.

Important Tips Statement 3 is correct. The concept of 'Rule of Law' has the
following three elements or aspects:
Other rights that are implicit to right to freedom of speech
are: Absence of arbitrary power, that is, no man can be
punished except for a breach of law.
Freedom of the press.
Equality before the law, th~t is, equal subjection of all
heed om of commercial advertisements.
citizens to the ordinary law of the land administered hy
Right agalml lappi11g uf Lde!Jhonic conversation. the ur<liuary law t-uurts.
Right to telecast, that is, government has no monopoly The primacy of the rights of the individual, that is, the
on electronic me<lia. constitulion i, Lhc result of Lhe rights of the individual
Right against bandh called by a political party or rather than the constitution being the source of the
organization. individual rights.
Right to know about government activities. 124. Solution (b)
Freedom of silence. Exp) Option b is the correct answer.
Right against imposition of pre-censorship on a Article 17 of Indian Constitution abolishes 'untouchability'
newspaper. and forbids its practice in any form.

Dlll,---------------------------------------------,-Practicebook

KING R QUEEN P [ऋषभ राजपूत ]


II INDIAN POLITY AND GOVERNANCE II
Statement l is incorrect: The term 'untouchability' has Statement l is incorrect. Martial Law is imposed to restore
not been defined either in the Constitution or in the Act. the breakdown of law and order due to any reason. Martial
However, the Mysore High Court held that the subject matter law is imposed under extraordinary circumstances like
of Article 17 is not untouchability in its literal or grammatical war, invasion, insurrection, rebellion, riot or any violent
sense but the 'practice as it had developed historically in the resistance to law.
country'. Statement 2 is incorrect. The Supreme Court held that the
Statement 2 is correct: According to the Representation of declaration of martial law does not ipso facto result in the
the People Act 1951, a person will be disqualified for election suspension of the writ of habeas corpus. However, during
to the Parliament or state legislature if he convicted of an the operation of martial law, the military authorities impose
offence punishable under the Protection of Civil Rights Act, restrictions and regulations on the rights of the civilians, can
195This Act provides for punishment for the preaching punish the civilians.
and practice of "untouchability", and for the enforcement Statement 3 is incorrect. 'Martial law' is not defined
of any disability arising therefrom. anywhere in the Constitution. It means 'military rule'.
--
125. Solution (b) Important Tips
Exp) Option bis the correct answer Comparison between National emergency and Martial
Statement l is incorrect: The right to property is a law:
-~-~----·-
Constitutional as well as legal right but not a Fundamental National Emergency Martial Law
Right. Originally, the Constitution provided for seven 1l affects not only It affects only Fundamental
Fundamental Rights. However, the Right to Property (Article Fundamental Rights but Rights (PRs)
31) was dclctccl from the list of Pun<lamental Rights by the also Centre-state relations. ,_., .....
44th Amendment Act of 197H. Tt is m~de a legal right under Government and ordinary Government and ordinary
Article 300-A in Part XII of the constitution. law courts continue. law courts suspended.
Statement 2 is correct. The Right to Property protects National Emergency can Martial law is imposed to
private property against executive action but not against be imposed only on three restore the breakdown of
legislative action. It can be regulated i.e. curtailed, abridged grounds- war, external law and order due to any
or modified without constitutional amendment by an aggression and armed reason
ordinary law of the Parliament. rebellion
1---------.-.----··-

Statement 3 is correct. In case of violation, the aggrieved It is imposed either in the It is imposed in some
person cannot directly move the Supreme Court under entire country or in any part specific area of the country
Article 32 (right to constitutional remedies including writs) of it.
--
for its enforcement. He can move the High Court under Has specific & detailed It has no specific provision
Article 226. provisions in the in the Constitution. It is
Constitution. It is explicit. implicit.
126. Solution (a)
Exp) Option a is the correct answer. 128. Solution (a)
Statement I is correct: There are 2 limitations imposed Exp) Option a is the correct answer.
as 'reasonable restrictions' to the exercise of Freedom The procedure established by Law would only assess
of Movement guaranteed under Article 19 of Indian whether the legislature has followed the right procedure to
constitution: form a law and whether executive has implemented the law
interests of general public in correct manner.
interests of any schedulcd Lrihc. Under due process of law, the court would not only check
Statement 2 i& incorrect: The freedom to leave lhc borders the procedurnl vuli<lity but also .::heLk. whether a Jaw is Just,
of India as per an individual's will and return back, ceiling fair and reasonable.
a citizen have been covered under the expanded scope of Statement 1 is correct: The term 'procedure established by
Article 21, by judicial activism in Maneka Gandhi case, law' has been explicitly mentioned in the India Constitution.
1978. Freedom of Movement under Article 19 covers the Article 21 states that no person shall be deprived of his
freedom of Indian citizens to move freely within India only. life or personal liberty except according to the procedure
established by law.
127. Solution (d)
Statement 2 is incorrect: The term 'due process of law'
Exp) Option d is the correct answer. does not imply that laws are made/ are to be made by
Article 34 provides for the restrictions on fundamental the _judiciary. It simply tests the li1ws on the grounds of
rights while martial law is in force in any area within the reasonableness and fairness. It aims to protect individuals
territory of India. However, the expression 'martial law' has against the arbitrary actions of both the executive and the
not been defined anywhere in the Constitution. legislature.

----111

KING R QUEEN P [ऋषभ राजपूत ]


II INDIAN POLITY AND GOVERNANCE II
129. Solution (a)
Important Tips
Exp) Option a is the correct answer. The Doctrine of Ancillary Powers implies that the
The Fundamental Rights are enshrined in Part III of the power to legislate on a topic of legislation carries with it
the power to legislate on an ancillary matter which can
Constitution from Articles 12 to 3They are not absolute but
be said to be reasonably included in the power given.
qualified. The state can impose reasonable restrictions on
The Doctrine of Severability states that the whole
them.
law or act would not be held invalid, but only those
Statement l is incorrect: Fundamental rights can be provisions will be held invalid which are inconsistent
amended by the Parliament by a constitutional amendment with the fundamental rights.
but only if the amendment does not alter the basic structure This Doctrine of Eclipse is applied when any Act/ law
violates fundamental rights. Using this doctrine, the
of the Constitution.
court makes the entire law unenforceable. Since the
Statements 2 is incorrect: Fundamental Rights are Fundamental Rights overshadows the law, it is called
guaranteed by the Constitution to all persons without the doctrine of eclipse.
any discrimination. The Fundamental Rights are meant
132. Solution (b)
for promoting the ideal of political democracy and not
Exp) Option b is the correct answer.
limiting it.
Article 26 of the Indian Constitution deals with Freedom
Statements 3 is correct: The Fundamental Rights are to Manage Religious Affairs. This Fundamental Right
guaranteed and protected by the Constitution of India. deals with the rights of religious denomination or any of its
They are 'fundamental' also in the sense that they are most sections.
essential for the all--round development of individuals. Statement l is correct: Article 26 grants following rights to
the religious denominations (or its sections):
130. Solution (a) 1. Right to establish and maintain institutions for religious
Exp) Option a is the correct answer. and charitable purposes.

Liberals uphold the principle of competition as the 2. Right to manage its own affairs in the matters of religion.

most efficient and fair way of distributing resources and 3. Right to own or acquire immovable property.
rewards in society. They believe that while states may have 4. Right to administer such property in accordance with
the law.
to intervene to try and ensure a minimum standard of living
Statement 2 is incorrect: The Constitution does NOT
and equal opportunities for all, this cannot by itself bring
define the term religious denomination.
equality and justice to society. Competition between people
Statement 3 is incorrect: The rights granted under Article
in free and fair conditions is the most just and efficient way
26 are not subjected to any other Fundamental Rights. In
of distributing rewards in a society. other words, they are not subordinate to other Fundamental
Rights.
Important Tips
Statement 4 is correct: The Constitution explicitly provides
Socialism advocates some kind of government
regulation, planning and control over certain areas that the rights granted under Article 26 are subject to public
order, health and morality.
such as education and healthcare.
Unlike Socialists, liberals do not believe that political, Important Tips
economic and social inequalities are necessarily linked. The term religious denomination has been defined by the
Supreme Court of India in the Shinu- Mutt case. A religious
131. Solution (d) denomination should satisfy the following conditions
Exp) Option d is the correct answer. It should be a collection of individuals sharing beliefs
Option dis correct: The Doctrine of Colorable Legislation is for their spiritual well-being.
used when the legislature does not possess powers to make It should have common organization.
law 011 a particular subject but indirectly makes one. It is It should be designated by a distinctive name.
also called 'Fraud on the Constitution'. The doctrine traces 133. Solution (a)
its origin to the Latin maxim that, "whatever legislature Exp) Option a is the correct answer.
cannot do directly, it cannot do indirectly". The doctrine Article 28 deals with the Freedom of Attending Religious
was used by the Supreme Court of India in the case of Instruction.
Gajapati Narayan Deo v. The State of Orissa to decide legality Statements 1 and 2 are incorrect: Religious instruction can
of the statute enacted. be allowed in an institute recognized by the State and an

KING R QUEEN P [ऋषभ राजपूत ]


II INDIAN FOUTY AND GOVERNANCE
II
institute receiving aid from the State. However, it shall be Option 4 is incorrect: Right against Untouchability has
allowed on a voluntary basis, and nobody can be forced to been provided under Article 17 of Part HI of the Indian
parlicipaie in the religious activities or instructions in thal Constitution.
institute.
Important Tips
Statement 3 is incorrect: Religious instruction is not
Other important rights outside Part III of the Constitution
in the institute which is administered by the
are:
State but has been established under any endowment or
trust which requires that religious instruction shall be Article 300-A (Part XII): No person shall be deprived
imparted in such institution. of his property except by the authority of the law.
Statement 4 is correct: There is absolute prohibition in Article 301 (Part XIII): Trade, commerce and
imparting religious instruction in the institutes wholly intercourse throughout the territory of India shall be
maintained out of the State Funds. Hence, the given statement free.
is correct.
136. Solution (b)
134. Solution (c) Exp) Option b is the correct answer.
Exp) Option c is the correct answer. Statement l is correct: It is true that laws enacted by both
Article 32 provides for the 'Right to the Constitutional the Parliament and State legislative assembly can be struck
Remedies' for the enforcement of fundamental rights. down by the High court if it violates fundamental rights.
Thus, an aggrieved citizen for violation of his Fundamental Article 226 of the Indian constitution provides the high
Rights can approach Supreme Court of India. Dr. Ambedkar court and Article 32 provides the supreme court - the power
has called this Article as the 'heart and soul' of the Indian of judicial review.
Constitution. Statement 2 is correct: Article 13 uses wider definition of
Statement l is incorrect: The Constitution of India neither laws, as per article 13 constitutional amendment act too
defines nor mentions the term 'basic structure'. Article is a law. Thus, any constitutional amendment act which
32 forms the basic structure, but it is not declared in the contravenes the provisions of fundamental rights can be
Constitution itself. struck down by the Supreme court and High court.

Statement 2 incorrect: The Article does not empower Statement 3 is incorrect: As per Article 13, a law includes
a custom or social practises having the force of law. The
Supreme Court with the exclusive jurisdiction over issuing
Supreme Court can ban those social practices which
writs. As per this Article, Parliament can empower any
contravenes fundamental rights.
other Courts to issue directions, orders and writs of all
kinds. Statement 4 is incorrect: Executive actions by civil servants
which violate fundamental rights can be struck down by the
Statement 3 correct: Writ jurisdiction in India has been
Supreme Court. As per Article 13, a law includes delegated
borrowed from the English Common Law. They are called
legislation such as order, bye law, rules and regulations.
'prerogative writs' in England.
Statement 4 is incorrect: As per Article 32, the right to move 137. Solution (d)
Supreme Court cannot be suspended unless otherwise stated Exp) Option d is the correct answer.
in the Constitution. Article 359 states that President can Article 19 of the Indian Constitution deals with the Right to
suspend the right to move any court for the enforcement of Freedom of Speech and expression.
Fundamental Rights during National Emergency.
Statement l is incorrect: The Supreme Court, in K.S.
135. Solution (c) Puttaswamy case, held that the right to privacy is protected
Exp) Option c is the correct answer. as an intrinsic part of the right to life and personal liberty
under Article 21 (not under Article 19).
All Fundamental Rights are Constitutional Rights, but
Statement 2 is incorrect: As per the Gujrat High Court, life
vice versa need not be true.
and liberty under Article 19 and 21 of a foreigner cannot
Option l is correct: Article 326 in part XV deals with adult be invoked on his behalf when the individual himself is
suffrage. It states that elections to the Houseof the People not in India. Also, the rights under Article 19 are available
and to the Legislative Assemblies of States to be on the basis only to the citizens of India and not to foreigners.
of adult suffrage.
Option 2 is correct: Article 265 in Part XII states that no 138. Solution (b)
tax can be levied except by the authority of law. In other Exp) Option b is the correct answer.
words, levying of any tax must be based on a statute and it The Americans adopted the Bill of Rights into their
can only be imposed by an authority authorized under as Constitution in the form of 10 amendments in 179While,
such under a statute. Part III of the Indian Constitution contains the Fundamental
Option 3 is incorrect: Right to free legal aid falls under Rights (Articles 12-35).
Article 21 of the Part HI of the Constitution which provides Statement 1 is incorrect: In the United States, no person's
Right to Life and Personal Liberty. life and liberty can be deprived without following the due

Practicebook---------------------------------------------,DJ

KING R QUEEN P [ऋषभ राजपूत ]


II INDIANFOL:ITYANDGOVERNANCE II
process of law, in India on the other hand the life and 141. Solution (a)
persona.I liberty can be taken away only according to the Exp) Option a is the correct answer.
procedure established by law. Only in case of depriving
Statement 1 is correct: According to Article 20, No person
someone of the Right to Life, the Supreme Court introduced
accused of any offence shall be compelled to be a witness
the concept of due process of law in Maneka Gandhi Case.
against himself. An individual cannot be compelled to be a
Statement 2 is correct: In the United States, the First witness in his own case by giving oral or written evidence.
Amendment guarantees press freedom, whereas in India, it The Code of Criminal Procedure also gives the defendant the
is implicit in Article 19(l)(a), which guarantees freedom of right to remain silent.
speech and expression.
Statement 2 is incorrect: The right against self-incrimination
Statement 3 is correct: The right to property was abolished
is available only during criminal proceedings and not during
as a fundamental right in India in 1978, but it is still a
civil proceedings or the proceedings which are not criminal
fimdamental right in the United States and as per the
in nature.
bill of rights, no property can be taken away without just
compensation. Statement 3 is incorrect: The protection under Article 20
does not extend to:
139. Solution (b) 1. Compulsory production of material objects
Exp) Option b is the correct answer. 2. Compulsion to give blood specimen, thumb impression,
State1m:nt l is incorrect: In the State of Madras vs. VG specimen signature.
Row, it has been held that the test of reasonableness should 3. Compulsory exhibition of body
be appliied in the context of each statute concerned and no
abstrac1t standard or general pattern of reasonableness can Important Tips
be laid down as applicable to all the cases. Article 20 of the Part III of the Indian Constitution deals
State1m,nt 2 is correct: In the State of UP vs. Kaushailiya, with 'Protection in Respect of Conviction for Offences'.
Supreme Court has held that prevailing social values and The Article covers three rights-
social needs which are intended to be satisfied by the No ex-post-facto law: According to this, a person shall
restrictions also help in deciding 'reasonableness' of a law. be convicted of any offence, except for the violation of
a law in force during the commission of an offence. It
Important Tips also provides that no penalty can be levied which is
Other considerations in deciding reasonableness can be: greater than the one prescribed under the law in force
The Directive Principles of State Policy (DPSP) at the time of commission of the offence
Arbitrariness and excessiveness of the restrictions No double jeopardy: A person cannot be punished/
penalized more than once for the same offence
Balance between restrictions imposed and the social
control envisaged. No self-incrimination: A person cannot be compelled
Proximity between restrictions and the objects to be to be a witness against himself.
achived. 142. Solution (b)
140. Solution (b) Exp) Option bis the correct answer
Exp) Option b is the correct answer. The First Amendment Act, 1951 inserted Articles 31A and
31B to the Constitution. Article 31C was inserted in the
Statement l is incorrect: In Kesavananda Bharati case
Constitution by 25th Amendment Act, 1971.
(19'73 ), Supreme Court held that the Parliament could
amend the fundamental rights without altering the 'basic Statement 1 is correct: Article 31A saves five categories of
si:ructm:e' of the Constitution. Thus, the parliament does laws from being challenged and invalidated on the ground
not have unlimited power to amend the constitution. of contravention of the fundamental rights conferred by
Article 14 and Article 19. It includes Acquisition of estates
Statement 2 is correct: In the Golaknath v. State of Punjab, and related rights by the State, Taking over the management
the Supreme Court held that parliamentary powers under of properties by the State, Amalgamation of corporations,
Article 368 were not absolute, and that the Parliament Extinguishment or modification of rights of directors
cannot abridge fundamental rights as they are included or shareholders of corporations and Extinguishment
um:l.er Part HI. or modification of mining leases. It also provides the
Statement 3 is correct: Supreme Court, in Minerva Mills guaranteed right to compensation in case of acquisition or
Case, upheld the validity of Article 31 C. According to Article requisition of the private property by the state.
31 C, a law that seeks to implement any or all of the directive Statement 2 is incorrect: Article 3 lB protects the acts and
principles of Part IV shall not be deemed invalid just because regulations included in the Ninth Schedule from being
it violates a citizen's fundamental rights under Article 14 challenged and invalidated on the ground of contravention
(equality before the law) or Article 19. Hence, fundamental of any of the fundamental rights. The scope of Article 31 B is
rights can be amended to give effect to directive principles. wider than Article 3 lA as it immunizes any law included in

KING R QUEEN P [ऋषभ राजपूत ]


II INDIAN POLITY AND GOVERNANCE II
the Ninth Schedule from the Fundamental Rights (unlike Pair l is correct: It protects an individual ag:ii11st unhwful
article 31A that protects only five categories). However, imprisonment. A writ of habeas corpus means that the court
the Supreme Court in its judgement in the I.R. Coelho case orders that the arrested person shouJd he presented bd<lre ii.
(2007) ruled that even laws under the Ninth Schedule would It can also order to set free an arrested person if the manner
be open to scrutiny if they violated Fundamental Rights or or grounds of arrest are not lawful or sat'isfactory.
the basic structure of the Constitution. Pair 2 is incorrect: Mandamus writ is issued when the court
Statement 3 is correct: Article 31 C says that no law finds that a particular office holder is not doing legal duty
that seeks to implement socialistic directive principles and thereby is infringing on the rights of an individual.
specified in Articles 39 (b) and 39(c), shall be declared void Pair 3 is incorrect: Prohibition is the writ issued by a higher
on the grounds of contravention of the fundamental rights court (High Court or Supreme Court) when a lower <:ourl
conferred by Article 14 or Article 19. has considered a case going beyond its jurisdiction.
143. Solution (a) Pair 4 is incorrect: Under Certiorari writ, the court orders a
Exp) Option a is the correct answer lower court or another authority to transfer a matter pending
before it to the higher authority or court. Whereas Qno- War-
Statement l is correct: Article 18 abolishes titles and makes
ranto means 'by what authority or warrant'. It prevents illegal
provisions in that regard. The hereditary titles of nobility
usurpation of public office by a person.
like Maharaja, Raj Bahadur, Rai Bahadur, Rai Saheb, Dewan ·- -- --· _____ _,

Bahadur, etc., which were conferred by colonial States are Important Tips
-·~·--· -····
banned by Article 18 as these are against the principle of Writ Jurisdiction of Writ Jutisdkt ion of High
equal status of all. Supreme Court Court
Statement 2 is incorrect: According to article 18(c), a The Supreme Court can A high cour·c can issue
foreigner holding any office of profit or trust under State only issue writs for the writs for ar iy purpose,
cannot accept any title from any foreign state without the enforcement of fundamental including the enforcement
consent of the President. No such bars for foreigners just rights. of fundamental
residing in the country.
The Supreme Court may A high court may issue
Statement 3 is incorrect: It provides that no citizen or issue writs against a person writs only against a
foreigner holding any office of profit or trust under the State or government throughout person residin g within its
is to accept any present, emolument or office from or under India's territory. territorial jur isdiction or
any foreign State without the consent of the president. against a go, ·crnmcnt or
144. Solution (d) authority locat,,d ou.tside its
territorial juris diction if the
Exp) Option d is the correct answer.
cause of action arises within
The Supreme Court held that the freedom of speech and its territorial
.....
expression includes the following:
A remedy under Article A remedy undf r Article 226
(a) Right to propagate one's views as well as the views of 32 is a Fundamental Right is discretionar y, and thus
others, in and of itself, and thus a high court 1l my refuse to
(b) Freedom of the press. the Supreme Court cannot exercise its writ jurisdici.ion.
(c) Freedom of commercial advertisements. refuse to exercise its writ
jurisdiction.
( d) Right against tapping of telephonic conversation.
( e) Right to telecast, that is, government has no monopoly
on electronic media. 146. Solution (a)

(f) Right against bandh called by a political party or Exp) Option a is the correct answer.
organisation. Freedom or Liberty, as elaborated in the Preamble, is very
(g) Right to know about government activities. essential for the successful functioning of the Indian demo-
(h) Freedom of silence. cratic system. Absence of constraints is only one dimension
of freedom. Freedom is also about expanding the ability
(i) Right against imposition of pre-censorship on a
newspaper. of people to freely express themselves am! develop their
potential. Freedom in this sense is the condition in which
The Supreme Court has held that though there is Right to
people can develop their creativity and capabilities.
demonstration or picketing but not right to strike.

145. Solution (a) 147. Solution (b)

Exp) Option a is the correct answer. Exp) Option b is the correct answer.
The Supreme Court (under Article 32) and the high courts Article 15 provides that the State shall not discrirn i nate
(under Article 226) can issue the writs of habeas corpus, against any citizen on grounds only of religion, race, caste,
mandamus, prohibition, certiorari and quo-warranto. sex or place of birth.

Practicebook----------------------------------------- -------IJI
mrom m~
KING R QUEEN P [ऋषभ राजपूत ]
II INDIAN POLITY AND GOVERNANCE II
Statements 2, 3 and 5 are corirect: There are four exceptions Important Tips
to the general rule of non-discrimination mentioned under
Article 350 A:
Article 15 of Indian Constitution.
It says there shall be a Special Officer for
The state is permitted to make any special provision for minorities to be appointed by the President.
women and children. For example, reservation of seats "lt shall be the duty of the Special Officer to investigate
for women in local bodies or provision of free education all matters relating to the safeguards provided for
linguistic minorities under this constitution and report
for children.
to the President those matters."
The state is permitted to make any special provision
149. Solution (d)
for the advancement of any socially and educationally
backward classes of citizens or for the scheduled castes Exp) option d is the correct answer.

and scheduled tribes. For example, reservation of seats Statement 1 is correct: Governor of states enjoy some of im-
munities from the rule of equality before law. Some of these
or fee concessions in public educational institutions.
immunities are
The state is empowered to make any special provision for
He is not answerable to any court for the exercise and
the advancement of any economically weaker sections performance of the powers and duties of his office.
of citizens. No criminal proceeding shall be instituted or continued
Statements l and 4 are incorirect: Constitution under Arti-- against him during his term of office.
de 15 does not provide any exclusive exception for the Spe- No process for arrest or imprisonment of him from any
cial provision for transgender communities with extended court during his term of office,
scope and Special provision for physically disabled persons. No civil proceedings shall be instituted during his term
of office in any court in respect of any act done by him in
Important Tips
his personal capacity, whether before or after he entered
Constitution (103rd Amendment) Act, 2019 upon his office, until the expiration of two months next
It introduced economic reservation by amending after notice has been delivered to him
Articles 15 and 16. It inserted Article 15 ( 6) and Article
Statement 2 is corirect: The foreign sovereigns (rulers), am-
16 (6) in the Constitution to allow reservation for the
bassadors and diplomates enjoy immunity from criminal and
economically backward in the unreserved category.
civil proceedings.
It was enacted to promote the welfare of the poor not
Statement 3 is correct: The UNO and its agencies enjoy dip--
covered by the 50% reservation policy for SCs, STs and
lomatic immunity.
Socially and Educationally Backward Classes (SEBC).
Statement 4 is correct: Members of parliament and state leg-
It enables both the Centre and the states to provide
reservation to the EWSs of society. islature also en Joy immunity from anything said or any vote
given by him in parliament/ state legislature or any commit-
148. Solution (b) tee thereof.
Exp) Option b is the correct answer. Important Tips
Article 29 and 30 of Indian constitution provides for the cul- Equality before law
tural and educational rights available to the citizens. Concept is of British origin

Statement l is corirect: Article 29 grants protection to both It states that no person (whether rich, poor, high or
low) is above law
religious minorities as well as linguistic minorities. However,
the Supreme Court held that the scope of this article is not It advocates equal subjection of all persons to the
ordinary law of the land administered by ordinary law
necessarily restricted to minorities only. This is because of
courts
the use of the words 'section of citizens' in the Article that
It is a negative concept as it restricts state from engaging
include minorities as well as majority. discrimination between individuals.
Statement 2 is correct: The Supreme Court held that the
150. Solution (c)
right to conserve the language includes the right to agitate
Exp) Option c is the correct answer.
for the protection of the language.
Constitution of India does not expressly provide the provi-
Statement 3 is incorrect: Article 30 grants certain rights to
sions related to the prisoners' rights but in the case of T. V.
minorities, whether religious or linguistic. Under Article Vatheeswaran v. State of Tamil. Nadu, it was held that the
30, all minorities shall have the right to establish and ad- Articles 14, 19 and 21 are available to the prisoners as well
minister educational institutions of their choice. as freemen. Prison walls do not keep out fundamental rights.

Practkebook

KING R QUEEN P [ऋषभ राजपूत ]


II INDIAN POLITY AND GOVERNANCE II
Option 3 fa correct; ArHde 14 of the Constitution of India body or an agency working as an instrument of the State
says that the State shall not deny to any person equality be-• falls within the meaning of the 'State' 1rn.de, Article 12. It
fore .!aw or the eq1rn1 protedic<tm oflaws within the territory is not mxessai-y that always a statutory authority should be
of India. engaged in performing governmental or sovereign functions.
Options 2 and 4 are correct: Article 21 of the Constitution Statement 2 is correct: All local authorities are included un-
of India says that No person shall he deprived of his life or der the definition of state under Article 12. According to List
personal liberty except according to procedure established II of VII Schedule 'local government' includes a munici-
by law. This indudcs right to speedy trial as a fondamcntal pal corporation, urban improvement trust, district boards,
right to the prisoners. mining settlement authorities and othn lo,~ 1 ;iuthorities
Option I. fa incorrect: Right to vote is not available to pris- for the purpose of local self-government or village admin-
<ilt.<'rn in Imlia ,Rl-i of yd. is1·r8't1nn
Statement J is incone.:t: In Atlit.ls, 17., tlw expression 'oth-
15L Sohlforn (d)
er authorities' is used after mentioning the Government,
Exp) Option cl is the correct answer.
Parliament of India, the Government and Legislature of each
Slakmeul I fa corn:d. The 86th Amendme1u Act of :!UU'.l, of the State and all local authorities. The Madras High Court
made elementary education a rumdamental Right under has held that 'other authorities' could only mean authorities
Artide 21 A. exercising governmental or sovereign functions. It cannot
Staten,,.ent 2 is coned. The 86th .Amendment Act of 2002 include persons, natural or juristic, such as, a university un-
changed the subject-matter of Artide 45. The amended less it is 'maintained by the State'.
directive principle requires the State to provide early chiJd ..
154. Solution (a)
hood care and education for all children until they complete
the age of six years. Exp) Option a is the correct answe.1',
Statement 3 is correct, The 86th Amendment Act of 2002 Preventive detention means detention of a person without
added the Eleventh Fundamental Duty which provides that trial. It is meant to prevent a person from committing any
the citizens should provide opportunities for education to crime. In other words, the person has not committed any
their child or ward between the age of six and fourteen years. crime yet, but the authorities anticipate him to be a threat to
luw und order.
152. Solution (b)
Statement l is incorrect: The Constitution provides that de-
Exp) Option b is the correct answer.
tention of a person cannot exceed three months. However,
The 86 th Constitutional Amendment Act made 'education in certain circumstances it can be extended if an Advisory
for all' a fundamental right. Board reports sufficient rnus;> for its extension.
Statement l is correct; The 86 th Amendment Act of 2002 Statement 2 is correct: Both the Centre and the States can
made 'Right to Education' a fundamental right. It inserted make laws on preventive detention. The Centre has exclusive
a new Artide 21-A under Part III of the Constitution which rights to make a law of preventive detention on the matters
provides that the "State shall provide free and compulso- connected with defense. foreign affairs Bn<i tlw s<>,urity of
ry educatim1 to aH children of the age ot six to fourteen India. On the matters pertaining lo security of the Stale,
years".
maintenance of public order and maintenance of supplies
Statement 2 is incorrect: The 86 th Amendment Act also and services essential to the community, both the Centre
changed the subject matter of Directive Principle of State and the States can concurrently enact laws.
Policy (DPSP) under Article 45, to provide that "the State
Statement 3 is incorrect: Protection under Article 22 re-
shall endeavor to provide early childhood care and educa-
garding preventive detention is applicable to both- Indian
tion for all children until they complete the age of six years".
citizens and the foreigners.
Hence, the DPSP pertains to only children up to 6 years of
age. 155. Solution (a)
Statement 3 is correct: The 86 th Amendment Act also added Exp) option a is the correct answer
a new Fundamental Duty under Article 51 A. It states that
Article 34 provides for the restrictions on fundamental
"it shall be the duty of every citizen of India who is a parent
rights while martial law is in force in any area within the
or a guardian to provide opportunities for education to his
territory of India.
child or ward between the age of 6-14 years". Hence, the giv--
en statement is correct. Statement l is incorrect: Martial law affects only Funda-
mental Rights (FRs) while National Emergency affects not
153. Solution (b) only Fundamental Rights but also Centre-state relations, dis-
Exp) Option bis the cm:r:;ct answer. tribution of revenues and legislative powers between Centre
Statement l is correct: The Supreme Court has held that and states. lt may extend the tenure of Parliament.
the term 'state' is wide c11ough to include all authorities on Statement 2 is incorrect: Martial law is imposed to restore
whom powers are conferred by law. Thus, even a private the breakdown of law and order due to any reason. While

I'racticebook---------------------------------------------alll

KING R QUEEN P [ऋषभ राजपूत ]


INDIANPOLITY·ANO GOVERNANCE II
National Emergency can be imposed only on three grounds- tion provides that all persons are equally entitled to free-
war, external aggression and armed rebellion. dom of conscience and right freely to profess, practice and
Statement 3 is coned: The definition of 'martial law' has propagate religion, subject to public order, morality and
not been mentioned explicitly in the Constitution of In- health and to the provisions of this part. Similarly, according
dia. 1'iowever, the provision of martial law is implicit in to provisions of UDHR, everyone has the right to freedom of
Article 34. Martial law literally means 'military rule'. It refers thought, conscience and religion.
to a situation where military authorities run the civil admin- Important Tips
istration as per their own rules and regulations. Ordinary
Universal Declaration of Human Rights
law is not adhered to when martial law is under operation.
It is a document that acts like a global road map for
156. Solution (c) freedom and equality - protecting the rights of every
Exp) Option c is the correct answer individual, everywhere.
Artide 15 of the Indian Constitution prohibits discrimi- The UDHR was adopted by the newly established
nation on grounds only of religion, race, caste, sex, or place United Nations on 10 December 1948, in response to
of birth. How,cver, there are four exceptions to this general the acts of mankind during the Second World War.
rule of non-discrimination. Although it is not legally binding, the protection of the
Option l is correct: As per Article 15 (4), the state is per- rights and freedoms set out in the Declaration has been
mitted to m.ake any special provision for the advancement incorporated into many national constitutions and
of any socially and educationally backward classes of cit- domestic legal frameworks.
izens or for the scheduled castes and scheduled tribes. For
instance, public educational institutions may offer fee dis- pirective Principles of State Policy
counts or seat reservations. This provision was added by the (DPSP)
first Amendrnent Act of J 95 l.
158. Solution: (a)
Option 2 is ,onect: The state is permitted to make any
Exp) Option a is the correct answer.
special proviBion for women and children. For example,
Following steps were taken by the Government of India to
reservation of seats for women in local bodies or provision
give effect to Directive Principles of state policies (DPSPs)
of free education for children.
Enactment of Cr PC (Criminal Procedure Code) in 1973
Important Tips give effect to Article 50 of DPSP i.e. it separated the
Affirmative action, also known as "positive discrimination;' judiciary from executive.
is a government policy that is designed to help minorities Passing Zamindari abolition bills give effect to Article
and disadvantaged groups in finding employment, getting 39(c) ofDPSP i.e. to prevent concentration of wealth and
admissions at universities, and obtaining housing. production.
Nationalisation of Commercial banks give effect to
Article 39(b) of DPSP i.e.to utilize the financial
Exp) Option c is the correct answer.
resources for promoting the common good.
lndia was a signatory to the Universal Declaration of Hu-
Provisions for reservation for the uplift of the ~chednled
man Rights ( 1948). Thus, great precaution was taken so that
castes give effect to Article 46 of DPSP i.e. to prnmote
Fundamental Rights as mentioned in Part 3 of Indian Con-
the educational and economic interest; of SC~, $Ts, and
stitlllion is co1.1c1n-rcnt with the provisions of the Universal
other weaker sections.
Declnnltion of Human Rights.
Creation of NALSA (National legal services Authorities
Option 1 is cnrrect: Article 21 of the Indian Constitution
Act) give effect to Article 39A of DPSP Le. to provide
provides that iw person shall he deprived of his life or per-
free legal aid to poor.
sonal liberty excepted according to procedure established by
law. Similarly, as per Universal Declaration of Human Rights Indian Penal Code was enacted in the British era in 1860
(UDI !R), everyone has the right to life, liberty and security much before the formation of Indian constitution and
Directive principles,
of prrson and 1m mw shall he subjected to iu-bitrary arrest,
detention or exile. 159. Solution: (d)
Oplinn 2 i.~ nm·cct: Article 23 (1) of the Indian Constitu- Exp) Option d is the correct answer
tion prohibits traffic in human being and 'Begar' and oth- Following provisions of Directive Principles of State Policy
er similar fonns of forced labour are prohibited, and any provides for social and economic justice. They direct the
contravention of th\., provision shall be an offence punish- state under:
able in accordnnce with law. Similarly, UDHR provides that Article 39 (c) to prevent the concentration of wealth and
no one shall he held in slavery or servitude and the slave means of production.
trade shall be prohibited in all forms, A1·ticle 43 to secure a living wage, a decent standard of
Option 3 is correct: Article 25 (l) of the Indian Constitu- life and social and cultural opportunities for all workers.

mForumm.18

KING R QUEEN P [ऋषभ राजपूत ]

You might also like